NUR 117 Pharmacology Final

¡Supera tus tareas y exámenes ahora con Quizwiz!

According to the guidelines of the Institute of Safe Medication Practices (ISMP) for proper use of abbreviations, symbols, and dose designations, what is the correct method of writing the abbreviations, symbols, and dose designations? Select all that apply. A. "4 mcg" instead of "4 ug." B. "10 cc" instead of "10 mL." C. "5.0 mg" instead of "5 mg." D. "30 u" instead of "30 units." E. "0.3 mg" instead of ".3 mg." F. "0.5 mcg" instead of "0.5mcg." G. "5000 units" instead of "5,000 units." H. "1,500,000 units" instead of "1.5 million units."

"4 mcg" instead of "4 ug." "0.3 mg" instead of ".3 mg." "0.5 mcg" instead of "0.5mcg."

Which statement made by a student nurse regarding syringes and needles indicates a need for further training? Select all that apply. Select one or more: A. "A 3-mL and a 5-mL syringe can measure ounces." B. "An insulin syringe has calibration lines that measure milliliters." C. "A 1-mL syringe has calibration lines that represent one-tenth of a milliliter." D. "A 6-mL syringe has the same calibration line measurements as the 5-mL syringe." E. "A special holder is required to administer medication when using a syringe cartridge."

"A 3-mL and a 5-mL syringe can measure ounces." "An insulin syringe has calibration lines that measure milliliters."

A nursing instructor asked the nursing student about the Institute of Safe Medication Practices (ISMP) guidelines for proper use of abbreviations, symbols, and dose designations. What answers given by the student indicate adequate learning? Select all that apply. A. "Spell out the word 'units.'" B. "Use a period after abbreviations." C. "Use the abbreviation 'microg' for microgram." D. "Use trailing zeros to indicate the dose of the drug." E. "Use the word 'million' for doses expressed in millions." F. "Use leading zeros for doses less than one measurement unit."

"Spell out the word 'units.'" "Use the word 'million' for doses expressed in millions." "Use leading zeros for doses less than one measurement unit."

A nursing instructor asks a nursing student about tall man lettering. Which statement made by the student nurse indicates adequate learning? Select all that apply. A. "Tall man lettering appears at the end of the drug name." B. "Tall man lettering appears in the middle of the drug name." C. "Tall man lettering is the use of mixed case letters in the drug name." D. "Tall man lettering is used to avoid confusion between classes of drugs." E. "Tall man lettering is used to avoid confusion between the similar sounding drugs."

**All answers are correct** "Tall man lettering appears at the end of the drug name." "Tall man lettering appears in the middle of the drug name." "Tall man lettering is the use of mixed case letters in the drug name." "Tall man lettering is used to avoid confusion between classes of drugs." "Tall man lettering is used to avoid confusion between the similar sounding drugs."

A health-care provider has ordered the administration of vancomycin 500 mg IV piggyback (IVPB) for a client. The IVPB available with the pharmacist is of premixed vancomycin 1 g with a total volume of 200 mL. What should the nurse consider to administer 500 mg of vancomycin to the patient? A. The nurse should use a different IVPB that contains only 500 mg of vancomycin. B. The nurse should administer 100 mL of the premixed IVPB to administer 500 mg of vancomycin. C. Either 1 or 2 D. None of above

**Two answers were taken** A. The nurse should use a different IVPB that contains only 500 mg of vancomycin. AND D. None of above

Which statement by a student suggests a need for further learning? A. "A drug label can have more than one dosage strength." B. "The dosage strength also includes the dosage form of the drug." C. "The dosage strength for liquid medications is usually expressed in mg/mL." D. "The dosage strength includes the strength of the medication with the unit of measurement."

**Two correct answers were taken** "The dosage strength also includes the dosage form of the drug." "The dosage strength includes the strength of the medication with the unit of measurement."

The primary health-care provider orders pyridostigmine intramuscularly (IM) 600 mcg for a 4-kg neonate born to a myasthenic mother. The dosage strength of the injection is 5 mg/mL. How many milliliters will the nurse administer to the neonate? (Round your answer appropriately per the rounding rules)

0.12mL

A nurse was asked to change the dose of a drug for the client from 150 micrograms to milligrams. What dose should the nurse administer to the client?

0.15 mg

A patient is prescribed levothyroxine 25 mcg by IV push. The drug on hand is levothyroxine sodium 100 mcg/mL. How many milliliters should you administer? (Round your answer appropriately per the rounding rules)

0.25mL

A client with an HIV infection is prescribed 300 grams of abacavir twice daily. The label on the drug bottle states 900 grams in 1 mL. Which volume of medication, in milliliters, should the nurse administer?

0.33mL

The medication order for an adult states, "Give 0.5 g of acetaminophen oral suspension PO every 12 hours." The acetaminophen in the medication bottle is available as 100 mg/mL. How many tsp will the nurse administer to the adult?

1tsp

The primary health-care provider orders hydroxyzine 0.05 g per day. The pharmacist sends hydroxyzine oral suspension of 25 mg/5 mL. The medication bottle comes with a measuring spoon that measures in teaspoons and tablespoons. How many teaspoons will the nurse instruct the client to take per day?

2 tsp

A patient with hyperthyroidism is prescribed an immediate dose of propylthiouracil (Propacil) 150 mg orally. The available drug is propylthiouracil 50 mg/tablet. How many tablets should you administer?

3 tablets

When checking a client's medication administration record, the nurse finds that 4 mg of loperamide needs to be administered. The loperamide label indicates that the dosage strength is 1 mg/7.5 mL. How many milliliters should the nurse administer?

30mL

An adult who has adrenal gland hyperfunction is prescribed to receive 2 g of mitotane (Lysodren) orally immediately. The drug on hand is mitotane 500 mg/scored tablet. How many tablets should you prepare to give?

4 tablets

According to the client's diet chart, 20 mL of gelatin is to be taken in the morning for breakfast. What should be the intake of the patient in teaspoons? Record your answer as a whole number.

4 tsp

A nurse has to administer 30,000 units of bacitracin. Per the drug label shown, if the nurse reconstitutes the product with 9.8 mL of sodium chloride, what volume (mL) of the solution will the nurse administer to the client? Round your answer to the tenth place if applicable. Do not forget to include the unit in your answer. **Image won't upload** One vial Bacitracin for intramuscular injection 50,000 units. Reconstitution with 9.8mL of sodium chloride injection containing 2 percent procaine hydrochloride will result in a concentration of 5,000 units per mL

6 mL

The health-care provider has ordered 30 mL of an antihistamine to be administered to an adult client if he or she experiences an allergic reaction after a blood transfusion. Which amount of the medication should the nurse administerin tsp?

6tsp

The pharmacy of a health-care facility sends a drug labeled 60 mg per tablet. The nurse administers 1.5 tablets. How many milligrams did the nurse administer?

90mg

A nursing instructor asks a nursing student about metric equivalents of household amounts. What answers given by the student indicate adequate learning? A. "1 ounce is equivalent to 30 mL." B. "1 teaspoon is equivalent to 10 mL." C. "1 tablespoon is equivalent to 20 mL." D. Both 1 and 3

A. "1 ounce is equivalent to 30 mL."

A patient is being discharged on enoxaparin (Lovenox). Which statement should you include in the discharge teaching plan? A. "An advantage of this drug is that labs do not need to be drawn to guide therapy." B. "Follow-up lab work must be drawn every 2 to 6 months." C. "Enoxaparin is given intramuscularly twice a day." D. "The purpose of this drug is to dissolve clots."

A. "An advantage of this drug is that labs do not need to be drawn to guide therapy."

Which side effect occurs when a patient is prescribed a DHT inhibitor? A. Dizziness B. Back pain C. Reduced fertility D. Stuffy nose

C. Reduced fertility

Which question is most important for you to ask a patient before administering a new drug? A. "Are you allergic to any drugs?" B. "Do you know what this drug is for?" C. "When was the last time you ate or drank?" D. "What other drugs have you taken in the last 24 hours?"

A. "Are you allergic to any drugs?"

Which precaution is most important to teach a patient who is prescribed any of the first-line drugs for generalized seizures? A. "Avoid drinking alcoholic beverages." B. "Take the drug with food or a snack." C. "Engage in regular exercise daily." D. "See your dentist twice a year."

A. "Avoid drinking alcoholic beverages."

A home care patient with chronic obstructive pulmonary disease (COPD) has oral candidiasis. What additional information should you obtain from this patient? A. "How often are you using your steroid inhaler?" B. "Do you share a toothbrush with any members of your family?" C. "When was the last time your inhaler drug prescriptions were filled?" D. "Have you taken any over-the-counter drugs for a cold or flu lately?"

A. "How often are you using your steroid inhaler?"

Which statement made by a patient prescribed drug therapy for glaucoma indicates more teaching is needed? A. "I am so glad my glaucoma has been cured with bimatoprost (Lumigan) in time to prevent blindness." B. "I will take care never to touch the tip of the applicator to my eye to prevent infection." C. "I will only call my doctor for eye redness if it lasts longer than just an hour or two." D. "I understand that I should wait for my vision to clear once the eye drops are in before I drive the car."

A. "I am so glad my glaucoma has been cured with bimatoprost (Lumigan) in time to prevent blindness."

You are teaching a patient about diuretic therapy. Which statement made by the patient indicates that more teaching is needed? A. "I am so thankful that my high blood pressure has been cured by this drug." B. "I always try to drink just about the same amount of fluid that I urinate each day." C. "I will call my health care provider if my heart rate is less than 60 beats/min." D. "I have been taking this drug early in the day so that I don't have to get up during the night."

A. "I am so thankful that my high blood pressure has been cured by this drug."

A patient is prescribed an oral corticosteroid for a chronic inflammatory health problem. Which precaution is most important to teach? A. "Never stop taking this drug without consulting your prescriber." B. "Avoid crowds and people who are ill." C. Be sure to take this drug with food." D. "Reduce your salt intake."

A. "Never stop taking this drug without consulting your prescriber."

You are teaching a patient about antihypertensive therapy. Which statement by the patient indicates the need for more teaching? A. "Now that my blood pressure is normal, I won't need to take my medication anymore." B. "When I take my blood pressure at home, I always try to take it at just about the same time every day." C. "I check the labels on cans and other food packages to be sure they do not have too much sodium." D. "I hope that by continuing to lose weight, I might not have to take medications to manage my high blood pressure."

A. "Now that my blood pressure is normal, I won't need to take my medication anymore."

A patient with a seizure disorder asks why bright spots are always seen before experiencing a seizure. What is your best response? A. "Some people experience a strange sensation called an aura before each seizure." B. "This is an unusual occurrence. I will notify your prescriber right away." C. "Bright spots before a seizure could indicate pressure in your brain from a tumor." D. "After the seizure do you feel confused, lethargic, and unable to respond to people?"

A. "Some people experience a strange sensation called an aura before each seizure."

You give the first dose of an antihypertensive drug to a hospitalized patient. Which instruction should you give to the patient next? A. "Stay in bed and call for help if you need to get up for any reason." B. "Urinate in a container so that we can keep track of your urine output." C. "If you develop a headache, we can give you some acetaminophen (Tylenol)." D. "You may get up but only if you need to use the bathroom."

A. "Stay in bed and call for help if you need to get up for any reason."

What is the most important precaution to teach a patient who is prescribed any antiviral drug? A. "Take the drug exactly as prescribed and for as long as prescribed." B. "Never take any antibacterial drugs while you are on antiviral therapy." C. "Drink at least 3 L of water daily for the entire time you are taking this drug." D. "Avoid unnecessary exposure to this drug by stopping it when you have no symptoms for 24 hours."

A. "Take the drug exactly as prescribed and for as long as prescribed."

Which response given by the student suggests a need for further A. "The ice chips melt to one-quarter of the amount fed." B. "Wound drainage is recorded as the output of the client." C. "The formula feeding is recorded as the input of the client." D. "The formula tube feeding may be continuous or intermittent."

A. "The ice chips melt to one-quarter of the amount fed."

The nurse begins infusing 1,000 D5W mL at 1455 to infuse over 9.5 hours. What will be the completion time of the IV? A. 0025 or 12:25 AM B. 0225 or 2:25 AM C. 1000 or 10:00 AM D. 1100 or 11:00 AM

A. 0025 or 12:25 AM

The nurse begins to infuse 75 mL of one-half NS IV at 0145 over 7 hr and 40 min. What is the completion time of the IV? A. 0925 or 9:25 AM B. 1015 or 10:15 AM C. 1045 or 10:45 AM D. 1110 or 11:10 AM

A. 0925 or 9:25 AM

The order is for benztropine 1 mg given once daily. The drug is available in 1 mg/mL. What is the time period over which the medication needs to be administered if the medication is administered at 1 mg/min? A. 1 minute B. 2 minutes C. 3 minutes D. 4 minutes

A. 1 minute

The primary health-care provider orders 25 mcg of a medication for a client. The medication has dosage strength of 75 mcg/mL. The nurse calculates the dosage volume to be 0.333 mL. Using the rounding rule, with which type of syringe should the nurse use to administer the medication? A. 1-mL B. 3-mL C. 5-mL D. 10-mL

A. 1-mL

The primary health-care provider has prescribed folic acid 1 mg/day for an adult with macrocytic anemia. What is the time period over which the folic acid needs to be administered if the rate of administration is 5 mg/min? A. 15 seconds B. 20 seconds C. 1 minute D. 1.7 minutes

A. 15 seconds

The order is for 1,200 mL D51/2 NS every 8 hours for a client. How many milliliters per hour will the nurse set on the IV pump? A. 150 mL/hr B. 2.5 mL/hr C. 125 mL/hr D. 100 mL/hr

A. 150 mL/hr

Which patient should always avoid taking sodium-containing antacids such as Alka-Seltzer and Bromo-Seltzer? A. 55-year-old female with heart failure B. 65-year-oldmale with overactive bladder C. 8-year-old child with Crohn's disease D. 25-year-old with leukemia

A. 55-year-old female with heart failure

A child weighing 30 kg has been prescribed a dosage of benazepril PO once daily. The drug label recommends 0.2 mg/kg per day. What is the safe weight-based recommended dose for the client? A. 6 mg/day B. 8 mg/day C. 9 mg/day D. 10 mg/day

A. 6 mg/day

A patient experiences facial angioedema after receiving a dose of a new antibacterial drug. Which assessment should be performed first? A. Airway adequacy B. Peripheral lung sounds C. Rate and depth of respirations D. Symmetry of respiratory movement

A. Airway adequacy

Which antifungal drug causes some degree of renal impairment in everyone who receives it? A. Amphotericin B (Amphocin) B. Terbinafine (Lamisil) C. Caspofungin (Cancidas) D. Fluconazole (Diflucan)

A. Amphotericin B (Amphocin)

What types of drugs prevent or control nausea/vomiting? A. Antiemetic drugs B. Antimotility drugs C. Antidiarrheal drugs D. Antipropulsion drugs

A. Antiemetic drugs

A patient tells you that she is allergic to the drug her health care provider just prescribed. What is your best first response or action? A. Ask her to describe the allergic response she had previously to this drug. B. Ask her whether she received the drug she was allergic to by mouth or by injection. C. Thank her for the information and ask the health care provider to prescribe something else. D. Tell her that the health care provider will also prescribe an antidote that will prevent an allergic response to the prescribed drug.

A. Ask her to describe the allergic response she had previously to this drug.

What is the major adverse risk when administering an enteral supplemental feeding to a patient through a feeding tube? A. Aspiration of tube feeding contents. B. Abdominal pain when given too fast. C. Toxicity from too many vitamins and minerals. D. Peritonitis if the bowel ruptures.

A. Aspiration of tube feeding contents.

Which common over-the-counter agent is an antiplatelet drug? A. Aspirin B. Tylenol C. Vitamin C D. Magnesium

A. Aspirin

You check a patient for pain relief 1 hour after administering 15 mg of morphine intramuscularly. The patient is sleeping and has a respiratory rate of 10 breaths/min. What is your best first action? A. Attempt to arouse the patient by calling his or her name and lightly shaking the arm. B. Administer oxygen by mask or nasal cannula and notify the prescriber. C. Check the patient's oxygen saturation and raise the head of the bed. D. Document the finding as the only action.

A. Attempt to arouse the patient by calling his or her name and lightly shaking the arm.

A patient is prescribed intravenous (IV) adenosine (Adenocard) for treatment of supraventricular tachycardia. Which safety precaution is most important to perform before this drug is administered? A. Bring the crash cart and defibrillator to the patient's bedside. B. Make sure that all four bed side rails are in the upright position. C. Place the patient's bed in the lowest position. D. Continuously monitor blood pressure.

A. Bring the crash cart and defibrillator to the patient's bedside.

Which action or precaution is most important for before giving any type of drug for hypertension? A. Check the patient's blood pressure. B. Ensure that the patient is in a sitting position. C. Check the patient's pulse for regularity. D. Have the patient drink a full glass of water.

A. Check the patient's blood pressure.

What is the most important action for you to teach a patient to perform before taking his or her daily dose of digoxin (Lanoxin)? A. Check the pulse for a full 60 seconds. B. Look for edema in your ankles. C. Notify the prescriber for any heart palpitations. D. Take the drug at exactly the same time every day.

A. Check the pulse for a full 60 seconds.

You are teaching an older adult patient who has been prescribed famotidine (Pepcid) and his/her family about the drug. Which common side effect should you instruct the patient's family to watch for and report to the prescriber? A. Confusion B. Anxiety C. Depression D. Psychosis

A. Confusion

What types of over-the-counter drugs can worsen high blood pressure? A. Decongestants & Antihistamines B. Laxatives C. Cortisone creams D. Aspirin

A. Decongestants & Antihistamines

What is the most common side effect of antihistamines? A. Drowsiness B. Weight gain C. Difficulty breathing D. Excessive urination

A. Drowsiness

What is another term for seizure disorder? A. Epilepsy B. Enkephalin C. Narcolepsy D. Neuropathy

A. Epilepsy

Which female hormone supports pregnancy by directly maintaining the thickened uterine lining? A. Estrogen B. Progesterone C. Luteinizing hormone D. Follicle stimulating hormone

A. Estrogen

Deficiency of which nutrient during pregnancy can cause spina bifida in the unborn baby? A. Folic acid B. Chromium C. Ascorbic acid D. Cholecalciferol

A. Folic acid

You are to administer 20 mg of furosemide (Lasix) to a patient by the intravenous (IV) route. Which action is most important for you to take? A. Give the drug slowly over at least 2 minutes. B. Check the patient carefully for symptoms of low blood glucose levels. C. Mix the drug with potassium chloride to prevent a rapid drop in serum potassium levels. D. Monitor the IV site after giving the drug because furosemide causes severe tissue damage if infiltration occurs.

A. Give the drug slowly over at least 2 minutes.

Which blood laboratory test result indicates that the patient is maintaining overall good control of his or her diabetes A. Hemoglobin A1c is 5.2%. B. Ketone levels are negative. C. Fasting blood glucose level is 88 mg/dL. D. Glucose level immediately following a meal is 150%.

A. Hemoglobin A1c is 5.2%.

What administration technique should you use when giving a 2-year-old child ear drops? A. Pull the earlobe down and back. B. Pull the earlobe up and out. C. Keep the earlobe straight. D. Hang the patient's head over the side of the bed.

A. Pull the earlobe down and back.

Why is it important to maintain normal blood glucose levels in the body? A. High levels increase the risk for heart disease, strokes, blindness, and kidney failure. B. High levels increase the risk for seizure disorders, arthritis, osteoporosis, and bone fractures. C. Low levels increase the risk for peripheral neuropathy, Alzheimer's disease, and premature aging. D. Low levels increase the risk for obesity, pancreatitis, dehydration, and certain types of cancer.

A. High levels increase the risk for heart disease, strokes, blindness, and kidney failure.

Which condition is the most dangerous adverse effect of insulin? A. Hypoglycemia B. Hyperglycemia C. Severe hypotension D. Development of insulin allergy

A. Hypoglycemia

A patient is taking warfarin (Coumadin) daily to prevent blood clots from forming in deep veins. Which statement made by the patient indicates that more teaching is needed about this therapy? A. I have been eating more salads and other green leafy begetabl3s to prevent constipation. B. I have been using acetaminophen (Tylenol) instead of aspirin for pain C. Instead of a safety razor, I have been using an electric shaver D. In hot days, I make sure to drink at least two quarts of water.

A. I have been eating more salads and other green leafy begetabl3s to prevent constipation.

What is the intended response to an erythropoiesis-stimulating agent (ESA)? A. Increased red blood cell counts B. Decreased red blood cell counts C. Increased platelet counts D. Decreased platelet counts

A. Increased red blood cell counts

You are administering a patient's first dose of cyclizine (Marezine). A. Instruct the patient to call for help when getting out of bed B. Raise all 4 side rails to the upright position C. Give the patient a full glass of water with the medication D. Tell the patient to avoid eating for at least 2 hours

A. Instruct the patient to call for help when getting out of bed

A patient has been discontinued from warfarin (Coumadin) therapy for 3 weeks. Which laboratory test indicates that all effects of the warfarin have been eliminated? A. International normalized ratio (INR) of 0.9 B. Red blood cell count of 5 million/mm3 C. Hemoglobin of 16 g/dL D. Hematocrit of 42%

A. International normalized ratio (INR) of 0.9

What is the most common side effect of bile acid sequestrants? A. Intestinal gas B. Facial flushing C. Blurred vision D. Muscle weakness.

A. Intestinal gas

Why should parents be taught never to give bismuth subsalicylate (Pepto-Bismol) to a child? A. It contains aspirin that can cause Reye's syndrome B. It changes stool color making the identification of GI bleeding more difficult C. It has such powerful antidiarrheal effects that toxic megacolon can occur D. It produces a paradoxical reaction in children that results in worsening of the symptoms

A. It contains aspirin that can cause Reye's syndrome

Which statement regarding artificially acquired active immunity is true? A. It requires "boosting" on a regular schedule. B. It is present in only children and not in adults. C. It can be used for vaccinations but not for immunizations. D. It has the shortest duration of effectiveness of any type of immunity.

A. It requires "boosting" on a regular schedule.

Which assessment is most important to perform before administering a bronchodilator by inhalation? A. Listening with a stethoscope to the patient's breath sounds. B. Measuring blood pressure in the sitting and standing positions. C. Asking questions to assess the patient's cognition and mental status. D. Checking the oral mucous membranes for white or cream-colored patches.

A. Listening with a stethoscope to the patient's breath sounds.

Which class of diuretics is the most powerful? A. Loop diuretics B. Osmotic diuretics C. Thiazide diuretics D. Potassium-sparing diuretics

A. Loop diuretics

If a drug has been recalled from the market, which option in the drug label helps to trace the batch of the drug? A. Lot number B. Tall man lettering C. Black box warning D. Manufacturer's name

A. Lot number

Which side effect is caused by selective alpha-1 blockers for benign prostatic hyperplasia? A. Low blood pressure B. Male-pattern baldness C. Erectile dysfunction D. Urinary retention

A. Low blood pressure

Which electrolyte imbalance is most likely to trigger skeletal muscle spasms? A. Low serum calcium level B. High serum magnesium level C. Low serum potassium level D. High serum chloride level

A. Low serum calcium level

Which assessment is most important to perform before giving a patient who is prescribed levothyroxine (Synthroid) the first dose of the drug? A. Measuring heart rate and rhythm. B. Determining level of consciousness. C. Asking about an allergy to "sulfa" drugs D. Checking intake and output for the last 24 hours

A. Measuring heart rate and rhythm.

Which term means constriction of the pupil of the eye? A. Miosis B. Punctum C. Glaucoma D. Mydriasis

A. Miosis

What precaution is needed in managing the care of a patient with insomnia who also has liver disease? A. Monitor carefully for increased side effects and adverse effects. B. Teach the patient that a higher dose is needed for effective action. C. Check the patient's electrolytes before administering the drug. D. Assess the patient's gag reflex every hour.

A. Monitor carefully for increased side effects and adverse effects.

Why is it important to avoid killing off normal flora with antibacterial drugs? A. Normal flora can help provide protection against the development of pathogenic infections. B. Normal flora result in opportunistic infections while other bacteria result in pathogenic infections. C. When normal flora are not present, the immune system is suppressed, increasing the risk for infection. D. When normal flora are not present, the immune system is overactive, increasing the risk for autoimmune diseases.

A. Normal flora can help provide protection against the development of pathogenic infections.

For which patient should you question the prescriber about an order for the PDE-5 drug vardenafil (Levitra)? A. Patient with a history of cardiac disease who is also prescribed nitroglycerin for angina B. Patient with a history of type 1 diabetes and hypertension who is prescribed insulin and metoprolol C. Patient with a history of kidney stones and bladder infection who is receiving antibiotics D. Patient with a history of chronic obstructive pulmonary disease who is prescribed bronchodilators

A. Patient with a history of cardiac disease who is also prescribed nitroglycerin for angina

How do calcium channel blockers help to lower blood pressure? A. Relaxing arteries and slowing heart contractions B. Increasing constriction of muscles in veins C. Inducing weight loss and reducing obesity D. Increasing water and sodium loss

A. Relaxing arteries and slowing heart contractions

A patient is receiving magnesium by intravenous infusion. Which finding indicates that the patient's blood magnesium level may be too high? A. Respirations are 10 per minute and shallow. B. Heart rate is 66 beats/min and regular. C. The patient reports difficulty staying asleep. D. Episodes of diarrhea.

A. Respirations are 10 per minute and shallow.

A patient taking a thiazide diuretic has the following blood laboratory values for kidney function. Which value will you report to the prescriber immediately? A. Sodium 124 mEq/L B. Potassium 3.6 mEq/L C. Creatinine 0.9 mg/dL D. Blood urea nitrogen 16 mg/dL

A. Sodium 124 mEq/L

What is the main reason that insulin is needed as drug therapy for people who have type 1 diabetes? A. The beta cells of the pancreas no longer make insulin. B. The alpha cells of the pancreas no longer make insulin. C. The beta cells of the pancreas make too much glucagon. D. The alpha cells of the pancreas make too much glucagon.

A. The beta cells of the pancreas no longer make insulin.

What does the term "host" mean? A. The person infected by a virus. B. The type of virus known as a retrovirus. C. The type of virus known as a common virus. D. The type of infection that occurs only in immunosuppressed people.

A. The person infected by a virus.

A nursing student has titers drawn to show immunity to specific diseases prior to clinical. The Varicella blood antibody titer is reported as zero (0). What is the correct interpretation of this information? A. The woman has full immunity against shingles. B. The woman should receive the shingles vaccination. C. The woman is highly likely to develop shingles. D. The woman is at risk for developing chickenpox.

A. The woman has full immunity against shingles.

Why should a patient with ERD avoid substances such as chocolate, peppermint, alcohol, nicotine and caffeinated drinks? A. These substances increase the pressure in the lower esophageal sphincter and lead to reflux B. These substances decrease the pressure in the lower esophageal sphincter and lead to reflux. C. These substances increase the pressure in the duodenal sphincter causing reflux and emesis D. These substances decrease the pressure in the duodenal sphincter causing reflux and emesis

A. These substances increase the pressure in the lower esophageal sphincter and lead to reflux

A patient is prescribed spironolactone (Aldactone). Why should you advise the patient to avoid the use of salt substitutes? A. They increase a patient's risk for a high potassium level. B. They can increase the patient's risk for hypertension. C. They may lead to hypokalemia. D. They can cause water retention.

A. They increase a patient's risk for a high potassium level.

What type of fungal infections can be treated with topical antifungal agents? A. Those that are confined to the skin or mucous membranes B. Those that are located under the toenails or fingernails C. Those that are infected by only one specific type of fungus D. The type that only infect humans and not animals

A. Those that are confined to the skin or mucous membranes

Which drug for glaucoma is a beta blocker? A. Timolol (Timoptic) B. Travoprost (Travatan) C. Apraclonidine (Iopidine) D. Pilocarpine (Adsorbocarpine)

A. Timolol (Timoptic)

Why is it necessary to slow or prevent osteoporosis? A. To reduce the incidence of bone fractures. B. To reduce the need for calcium supplementation. C. To increase skeletal muscle strength and flexibility. D. To prevent excessive weight gain after menopause.

A. To reduce the incidence of bone fractures.

A patient reaches for the salmeterol (Serevent) inhaler with the onset of an asthma attack. What is your best instruction to the patient? A. Use the albuterol (Proventil) inhaler instead. B. Wait at least one full minute between puffs. C. Attach the spacer to the inhaler before using it. D. Take a deep breath, hold it for 15 seconds, and then exhale before using the inhaler.

A. Use the albuterol (Proventil) inhaler instead.

Which key point should you teach a pregnant woman who is prescribed opioids drugs during pregnancy? A. Your child may become addicted to opioids and go through withdrawal after birth. B. These drugs do not cross the placenta so are safe to use during pregnancy. C. Some opioids have been found to cause birth defects in animal studies. D. Opioids are not present in breast milk so it is safe to breastfeed.

A. Your child may become addicted to opioids and go through withdrawal after birth.

A patient receiving chemotherapy and prescribed ondansetron (Zofran) asks why the drug is given before meals. What is your best response? A. Zofran is given 30 minutes before your meals to prevent nausea. B. The purpose of Zofran is to move food rapidly through your GI tract C. Zofran works by preventing nausea caused by the morphine given for your pain D. If Zofran is given after your meals, food would interfere with its absorption.

A. Zofran is given 30 minutes before your meals to prevent nausea.

Which NSAID is a COX-2 inhibitor? A. celecoxib (Celebrex) B. ketorolac (Toradol) C. aspirin (Bufferin) D. ibuprofen (Motrin)

A. celecoxib (Celebrex)

Which drug suppresses thyroid activity? A. methimazole (Tapazole) B. metolazone (Mykrox) C. levothyroxine (Synthroid) D. liothyronine (Cytomel)

A. methimazole (Tapazole)

Which diuretic may cause an adverse effect of a higher than normal serum potassium level? A. spironolactone (Aldactone) B. bumetanide (Bumex) C. chlorothiazide (Diuril) D. furosemide (Lasix)

A. spironolactone (Aldactone)

Which question is most important to ask a patient who is prescribed acetazolamide (Diamox) before administering the first dose? A. "Do you have diabetes?" B. "Are you allergic to sulfa drugs?" C. "How long have you had glaucoma?" D. "Do you eat grapefruit or drink grapefruit juice?"

B. "Are you allergic to sulfa drugs?"

A nursing instructor asks a student nurse, "What do the red arrows symbolize from left to right in the insulin order shown?" What answer given by the student nurse indicates adequate learning? **Image won't show** Order as follows: Novolin N 21 Units subcut 2 AM A. "Brand name, insulin units to administer, route of administration, duration of administration" B. "Brand name, insulin units to administer, route of administration, frequency of administration" C. "Generic name, frequency of administration, route of administration, insulin units to administer" D. "Generic name, insulin units to administer, frequency of administration, route of administration"

B. "Brand name, insulin units to administer, route of administration, frequency of administration"

Which safety precaution should you teach a patient who is prescribed eye drops or eye ointment? A. "Wash your hands before and after applying the drug." B. "Do not share your eye medications with anyone else." C. "Avoid drinking alcoholic beverages while taking this drug." D. "Do not drive or use heavy equipment while your vision is blurred."

B. "Do not share your eye medications with anyone else."

A patient is prescribed intravenous (IV) acyclovir (Zovirax). Which question is most important to ask before giving the first dose of this drug? A. "Do you have a hearing problem or any trouble with your ears?" B. "Do you take medications for seizures?" C. "Are you allergic to sulfa drugs?" D. "Have you ever had asthma?"

B. "Do you take medications for seizures?"

A patient is prescribed intravenous ertapenem (Invanz). Which question is most important to ask before giving the first dose of this drug? A. "Do you have a hearing problem or any trouble with your ears?" B. "Do you take medications for seizures?" C. "Are you allergic to sulfa drugs?" D. "Have you ever had asthma?"

B. "Do you take medications for seizures?"

The prescriber orders a new drug over the telephone for a nursing home patient who has symptoms of a urinary tract infection. The order is for Gantanol, 2 g now and then 1 g every 12 hours for the next 10 days. What further information is most important for you to obtain from the prescriber? A. "How many refills are needed?" B. "Do you want the drug given orally or intravenously?" C. "Which brand of drug should be given, or is this a generic drug?" D. "Does this drug need to be given with a meal or on an empty stomach?"

B. "Do you want the drug given orally or intravenously?"

Which action is most important to teach a patient prescribed to take allopurinol (Zyloprim) daily for gout? A. "Avoid operating heavy machinery." B. "Drink plenty of water." C. "Avoid alcohol and caffeine." D. "Take the drug only when symptoms of gout are present."

B. "Drink plenty of water."

A patient who is very immunosuppressed is prescribed ketoconazole (Nizoral) for prevention of a fungal infection. Which statement made by this patient indicates the need for more teaching? A. "I will stop taking the drug if I should become pregnant." B. "I will stay in the sun as much as possible to avoid catching an infection." C. "I will avoid drinking alcoholic beverages during the time I am on this therapy." D. "If I miss a dose, I will take it as soon as I remember it and not wait until the next day."

B. "I will stay in the sun as much as possible to avoid catching an infection."

The RN is discussing patient-controlled analgesia (PCA) pumps with the student nurses. Which statements made by the student nurse is false regarding a PCA pump? A. "It is a specialized type of syringe pump." B. "It is an electronic infusion device that pumps a set volume of IV fluid." C. "It has preset limits on the amount and frequency of medication administration." D. "It allows the client to self-administer medication by pushing a button on the pump."

B. "It is an electronic infusion device that pumps a set volume of IV fluid."

A patient asks why an anticoagulant has been prescribed. What is your best response? A. "It will dissolve any clots in your blood vessels." B. "It will prevent any new clots from forming." C. "It will prevent a clot from migrating." D. "It will thin your blood."

B. "It will prevent any new clots from forming."

The patient with an elevated temperature is prescribed antibacterial therapy to be taken at home. The prescribed drug is amoxicillin (Amoxil) 250 mg three times a day. What would you teach the patient about dosing intervals for this drug? A. "Take the drug three times a day with meals." B. "Take the drug every 8 hours throughout the day." C. "Take the drug when you first get up, at lunch time, and with your evening meal." D. "Take the drug every 6 hours while you are awake."

B. "Take the drug every 8 hours throughout the day."

What should you teach a patient who has been prescribed potassium (K-Dur) tablets? A. "If you miss a dose, double your next dose to keep your blood level of potassium normal." B. "Take your potassium with food or a full glass or water to avoid nausea/vomiting." C. "Be sure to use salt su*bstitutes instead of salt so that your body will not retain water." D. "Eat lots of foods that are high in potassium such as bananas, spinach, broccoli, and sweet potatoes."

B. "Take your potassium with food or a full glass or water to avoid nausea/vomiting."

A 75-year-old patient tells you he is not planning to receive a "flu shot" this year because he had one just a year ago. What is your best response? A. "Because you are older and your immune system is more fragile, you should have one this year too as a booster." B. "The virus causing influenza often changes each year and a new influenza vaccination is needed every flu season." C. "The "flu shot" you had last year should still protect you for seasonal influenza, but you still need a vaccination for H1N1." D. "The fact that you have been vaccinated by injection just last year makes you a candidate to use the nasal vaccination this year."

B. "The virus causing influenza often changes each year and a new influenza vaccination is needed every flu season."

The patient is given a medication bottle with the drug labeled as 100 mg dose of metronidazole. What will be the dose in grams? A. 1 gram B. 0.1 gram C. 0.01 gram D. 0.001 gram

B. 0.1 gram

The primary health-care provider orders 1,000 mL D10W to be run at 2300 over 12 hours. What is the completion time of the IV? A. 1000 or 10:00 AM B. 1100 or 11:00 AM C. 2300 or 11:00 PM D. 0000 or 12:00 midnight

B. 1100 or 11:00 AM

A nurse needs to infuse 1 unit packed cells (240 mL) over 4 hours per order. The nurse has a blood administration set with a drop factor of 15 gtt/mL. What is the flow rate for this transfusion? A. 10 gtt/mL B. 15 gtt/mL C. 20 gtt/mL D. 25 gtt/mL

B. 15 gtt/mL

An adult client weighing 56 kg with bacterial infection is to receive aminoglycosides IV push (IVP) daily. The drug label recommends 5 mg/kg every 8 hr. What is the safe weight-based recommended dose for the client? A. 250 mg/day B. 280 mg/day C. 290 mg/day D. 300 mg/day

B. 280 mg/day

A patient is prescribed 300 mg of oral clopidogrel (Plavix) as a first dose. The drug on hand is clopidogrel 75 mg. How many tablets should you give? A. 3 tablets B. 4 tablets C. 2 tablets D. 3.5 tablets

B. 4 tablets

With which patient should you be prepared to administer glucagon as a drug? A. 60-year-old with type 2 diabetes who has renal impairment B. 42-year-old who has a blood glucose level of 25 mg/dL C. 28-year-old who has type 2 diabetes and is in diabetic ketoacidosis D. 10-year-old with a fasting blood glucose of 87 mg/dL

B. 42-year-old who has a blood glucose level of 25 mg/dL

An adult client with uterine bleeding has been prescribed 25 mg of estrogen. What is the time period over which the medication needs to be administered if the medication is administered at rate of 5 mg/min? A. 3 minutes B. 5 minutes C. 8 minutes D. 12 minutes

B. 5 minutes

Which fasting blood glucose level indicates that the antidiabetic drug prescribed for a patient with type 2 diabetes is effective at maintaining target blood glucose levels? A. 40 mg/dL B. 80 mg/dL C. 120 mg/dL D. 160 mg/dL

B. 80 mg/dL

Which statement about drug therapy for heart failure is true? A. The drugs work only directly on the heart muscle. B. 8Drug therapy improves heart function but does not cure heart failure. C. When heart function returns to normal, the drug therapy can be stopped. D. Drug therapy for heart failure does not work when the patient continues to smoke.

B. 8Drug therapy improves heart function but does not cure heart failure.

What problem is mst important to assess for in a patient who is prescribed loperamide (Imodium)? A. Tachycardia B. Abdominal distention C. Peripheral edema D. Respiratory crackles

B. Abdominal distention

A nurse needs to administer ranitidine hydrochloride to a hospitalized client with intractable duodenal ulcers. The prescription shows that 1 mL of Zantac is be administered. According to the instructions mentioned on the drug label depicted, which step must the nurse take? **Image won't upload** 2mL single-dose vial Zantac Injection 50mg (ranitidine hydrochloride) 25mg ranitidine/1 mL A. Administer 1 mL of Zantac solution and refrigerate for reuse. B. Administer 1 mL of Zantac solution and discard the remaining solution. C. Administer 1 mL of Zantac solution, reseal, and discard the bottle after 28 days. D. Administer 1 mL of Zantac solution and store the bottle at room temperature for reuse in the same patient.

B. Administer 1 mL of Zantac solution and discard the remaining solution.

What is the most important role of the health care worker in preventing drug errors? A. Always checking the patient's diagnosis before giving a drug B. Always following the "eight rights" of drug administration C. Being the one defense for detecting and preventing drug errors D. Being most likely to detect a drug error that has occurred

B. Always following the "eight rights" of drug administration

How are anticoagulant drugs and thrombolytic drugs different? A. There is no difference; they both have the same actions. B. Anticoagulants prevent clots from forming whereas thrombolytics can dissolve clots that have already formed C. Thrombolytics must be administered intravenously while all anticoagulants are administered as oral agents. D. Anticoagulants prevent clots by actually thinning the blood whereas thrombolytics reduce platelet aggregation and do not affect blood thickness.

B. Anticoagulants prevent clots from forming whereas thrombolytics can dissolve clots that have already formed

A child is receiving a seasonal influenza vaccination administered by nasal inhalation. What type of immunity is expected to develop as a result of this vaccination method? A. Artificially acquired passive immunity B. Artificially acquired active immunity C. Naturally acquired passive immunity D. Naturally acquired active immunity

B. Artificially acquired active immunity

A patient with heart failure is prescribed metoprolol (Toprol XL) 25 mg daily. The pharmacy sends metoprolol (Lopressor) 25 mg. What is your best action? A. Give the dose as provided because the drugs are the same. B. Ask the pharmacy to send the drug as ordered. C. Contact the prescriber for clarification. D. Check the patient's heart rate before giving the dose.

B. Ask the pharmacy to send the drug as ordered.

How are viral infections different from bacterial infections? A. Bacterial infections can be spread from one person to another, whereas viral infections cannot be spread directly. B. Bacterial infections can be cured by treatment with some anti-infective drugs, whereas viral infections are not cured by anti-infective therapy. C. Viruses only cause disease in a person who is immunocompromised, whereas bacteria can cause disease even among immunocompetent people. D. Viruses are the less mature form of a bacterium, so there is essentially no difference between viral infections and bacterial infections.

B. Bacterial infections can be cured by treatment with some anti-infective drugs, whereas viral infections are not cured by anti-infective therapy.

Which drug group for glaucoma can worsen asthma? A. Adrenergic agonists B. Beta blockers C. Cholinergics D. Prostaglandin agonists

B. Beta blockers

A patient is receiving an injection of Humulin 70/30 at 7:00 a.m. At what time should you expect the peak action for this drug? A. Between 7:30 a.m. and 8:00 a.m. B. Between 9:00 a.m. and 11:00 a.m. C. Between 1:00 p.m. and 5:00 p.m. D. Between 4:00 p.m. and 11:00 p.m.

B. Between 9:00 a.m. and 11:00 a.m.

Why do the most common side effects of any bile acid sequestrant include bloating, abdominal discomfort, and constipation? A. Many patients are lactose intolerand and these drugs contain lactose. B. Bile acid sequestrants exert their effects directly in the intestinal tract. C. The action of bile acid sequestrants on the liver releases bile into the intestinal tract. D. The drugs inhibit the absorption of dietary fiber, increasing its concentration and effects in the intestinal tract.

B. Bile acid sequestrants exert their effects directly in the intestinal tract.

What is the most serious adverse effect of HRT for menopause A. Pregnancy B. Blood clots C. Miscarriage D. Decreased mental concentration

B. Blood clots

What cardiac dysrhythmia can be helped or corrected by the drug atropine? A. Tachycardia B. Bradycardia C. Premature atrial contractions D. Premature ventricular contractions

B. Bradycardia

What action must you take for a diabetic patient who is prescribed niacin (Niacor)? A. Give insulin only in the presence of food. B. Check the blood glucose level more often? C. Decrease the morning insulin dosage. D. Encourage increased caloric intake.

B. Check the blood glucose level more often?

When you bring in the next dose of a drug that a patient first received 6 hours ago, the patient reports a "pounding" heart rate ever since taking the last dose. What is your best first action? A. Document the report as the only action. B. Check the patient's vital signs for changes. C. Hold the dose and notify the prescriber immediately. D. Reassure the patient that this is an expected response to the drug.

B. Check the patient's vital signs for changes.

A nurse is to administer a fractional portion of a total daily dose at specified intervals. Which type of dose is mentioned in the given scenario? A. Loading dose B. Divided dose C. Total daily dose D. Maintenance dose

B. Divided dose

What is the most important precaution to teach a patient who is on insulin therapy? A. Avoid sharing needles with family members. B. Do not skip meals when taking insulin. C. Always refrigerate your insulin. D. Avoid eating carbohydrates.

B. Do not skip meals when taking insulin.

A patient who has been self-injecting epoetin alfa (Procrit) three times weekly for anemia now reports all of these problems. For which one do you tell him or her to call 911? A. Fever and chills B. Drooping on the right side of the face C. 3lb weight gain over the past 2 weeks. D. Intermittent pain and redness at the injection site

B. Drooping on the right side of the face

Which hormone has an action opposite to the action of insulin? A. Glycogen B. Glucagon C. Hemoglobin A1c D. Adenosine triphosphate

B. Glucagon

Which food, drink, or herbal supplement should you warn a patient prescribed gemfibrozil (Lopid) to avoid? A. Caffeinated beverages B. Grapefruit juice. C. St. John's wort D. Dairy products

B. Grapefruit juice.

Which side effect is associated only with loop diuretics? A. Dizziness B. Hearing loss C. Urinary frequency D. Increased sun sensitivity

B. Hearing loss

What health problem is a possible adverse effect of any drug that reduces blood clotting? A. Pulmonary embolism B. Hemorrhage C. Thrombosis D. Diabetes

B. Hemorrhage

A patient is prescribed to start cyclobenzaprine (Flexeril). Before administering the drug, you find he has a slow and irregular heartbeat. What is your best action? A. Administer the drug as prescribed. B. Hold the dose and notify the prescriber. C. Apply a heart monitor before administering the drug. D. Substitute methocarbamol (Robaxin) for the Flexeril.

B. Hold the dose and notify the prescriber.

A patient who has just been prescribed estrogen-based hormone replacement therapy (HRT) also has all of the following health problems. For which problem should you check with the prescriber to determine whether the patient should take HRT? A. Asthma B. Hypertension C. Osteoporosis D. Diabetes mellitus

B. Hypertension

Where in or on the eye should eye drops be placed? A. Under the upper lid B. In a pocket of the lower lid (Conjunctival sac) C. Directly over the center of the eye D. In the corner of the eye closest to the nose

B. In a pocket of the lower lid (Conjunctival sac)

When is it acceptable to take a verbal order from the prescriber before giving a drug to a patient? A. During the nightshift when the prescriber is not at the hospital B. In an emergency situation such as a cardiac arrest C. When a patient is experiencing severe pain D. At any time it is convenient

B. In an emergency situation such as a cardiac arrest

A client is instructed to drink 14 ounces of water throughout the day. Which amount of water should the nurse provide to the client in mL? A. 310 mL B. 50 mL C. 420 mL D. 500 mL

C. 420 mL

Why is insulin only given by injection and not as an oral drug? A. Injected insulin works faster than oral drugs to lower blood glucose levels. B. Insulin is a small protein that is destroyed by stomach acids and intestinal enzymes. C. Insulin is a "high-alert drug" and could more easily be abused if it were available as an oral agent. D. Oral insulin has a high "first pass loss" rate in the liver and would require very high dosages to be effective.

B. Insulin is a small protein that is destroyed by stomach acids and intestinal enzymes.

A patient who has been prescribed lovastatin asks why all urine must be saved for intake and output measurements. What is your best response? A. All patients on this unit have orders for strict intake and output measurements. B. Intake and output measurements are important indicators of how well your kidneys are functioning. C. A side effect of this drug can be blockage of urine flow through the kidneys and decreased urine output. D. Sometimes this drug can cause the kidneys to make extra urine resulting in increased urine output and dehydration.

B. Intake and output measurements are important indicators of how well your kidneys are functioning.

Which type of nutrient deficiency should you assess for in a patient who eats mostly fresh fruit, red and yellow vegetables, rice, milk, and other dairy products? A. Vitamin A B. Iron C. Sodium D. Fluoride

B. Iron

Which bone or joint is most likely to be adversely affected by bisphosphonates? A. Eye orbit B. Jawbone C. Big toe D. Wrist

B. Jawbone

Which drug used to treat constipation is an osmotic laxative? A. Methylcellulose B. Lubiprostone C. Castor oil D. Ducusate

B. Lubiprostone

In which unit is the rate of primary IV infusion measured? A. Liters per hour B. Milliliters per hour C. Either of the above D. None of the above

B. Milliliters per hour

What is the main role of insulin in glucose (carbohydrate) metabolism? A. Conversion of complex carbohydrates into glucose B. Movement of glucose from blood into the cells C. Prevention of kidney excretion of glucose D. Prevention of the formation of fat cells

B. Movement of glucose from blood into the cells

A patient prescribed atorvastatin (Lipitor) reports all of the following problems or changes since starting this drug. Which problem or change do you report to the prescriber? A. Abdominal cramps and bloating B. Muscle aches and weakness C. Urinating more at night. D. Loss of taste for sweets

B. Muscle aches and weakness

Why are oral antidiabetic drugs not effective in the management of type 1 diabetes? A. These drugs are too powerful to give to children. B. Patients with type 1 diabetes do not produce insulin. C. The oral drugs are less predictable in lowering blood glucose levels. D. Patients with type 1 diabetes are more likely to have allergies to these drugs.

B. Patients with type 1 diabetes do not produce insulin.

A patient with which findings requires drug therapy for active tuberculosis (TB)? A. Negative TB skin test and chest x-ray but who has productive cough, fever, and shortness of breath B. Positive TB skin test, productive cough, and a cavitation on chest x-ray C. Positive TB skin test and a TB scar on chest x-ray D. Positive TB skin test as the only symptom

B. Positive TB skin test, productive cough, and a cavitation on chest x-ray

A patient taking a thiazide diuretic has the following blood laboratory values for kidney function. Which value will you report to the prescriber immediately? A. Sodium 136 mEq/L B. Potassium 2.6 mEq/L C. Creatinine 0.9 mg/dL D. Blood urea nitrogen 6 mg/dL

B. Potassium 2.6 mEq/L

Which type of insulin has the shortest duration of action? A. Short-acting insulin B. Rapid-acting insulin C. Intermediate-acting insulin D. Long-acting insulin

B. Rapid-acting insulin

A patient with active tuberculosis who has been taking isoniazid (INH, Nydrazid) and rifampin (RIF, Rifadin) reports having urine that is an orange color. What your best action? A. Obtain a specimen for culture and test the urine for occult blood. B. Reassure the patient that this is a normal drug side effect. C. Hold the dose and contact the prescriber. D. Document the report as the only action.

B. Reassure the patient that this is a normal drug side effect.

Which drug action is most likely to lower blood pressure? A. Increasing the effectiveness of heart pumping B. Relaxing (dilating) blood vessel smooth muscle C. Increasing the concentration of urine d. Retaining the body's concentration of potassium

B. Relaxing (dilating) blood vessel smooth muscle

What is the most common purpose of the international normalized ratio (INR)? A. Measuring the total amount of blood a person has B. Reporting the results of warfarin anticoagulation C. Assessing the number of clotting factors present D. Determining the number of platelets.

B. Reporting the results of warfarin anticoagulation

Which antitubercular drug causes the urine to turn reddish orange? A. Isoniazid (INH) B. Rifampin (RIF) C. Pyrazinamide (PAS) D. Ethambutol (EMB)

B. Rifampin (RIF)

Which substance should a patient avoid when prescribed a potassium supplement? A. Table salt B. Salt substitutes C. Garlic salt D. Onion salt

B. Salt substitutes

What must you teach any patient who is taking a diuretic drug? A. Avoid foods that are rich in potassium such as bananas and broccoli. B. Sit on the side of the bed for 1 to 2 minutes before getting out of bed. C. Notify the prescriber if the heart rate is less than 70 beats/min. D. Keep a record of dietary intake for a few weeks.

B. Sit on the side of the bed for 1 to 2 minutes before getting out of bed.

Which foods should you teach a patient who has a mild vitamin K deficiency to eat? A. Eggs and fortified milk B. Spinach and romaine lettuce C. Sunflower seeds and almonds D. Carrots, sweet potatoes, and pumpkin

B. Spinach and romaine lettuce

What should you teach a woman of childbearing age who is prescribed a statin drug? A. These drugs should not be taken during pregnancy, but are safe to use when breastfeeding. B. Statin drugs can interfere with fetal brain development and should not be taken during pregnancy or breastfeeding. C. Because these drugs control fats produced by the liver, they will help control weight gain during pregnancy. D. You can take these drugs during pregnancy as long as you do not have a history of muscle or liver problems.

B. Statin drugs can interfere with fetal brain development and should not be taken during pregnancy or breastfeeding.

Which statement about testosterone is accurate: A. Testosterone levels begin to decrease around the age of 50 years B. Testosterone helps maintain male sexual characteristics C. Too much testosterone leads to decreased body muscle mass D. Adequate red blood cell levels are essential for production of testosterone

B. Testosterone helps maintain male sexual characteristics

Which statement is correct regarding the drug label in the image? **Image won't upload** METHYLPREDnisolone 40 mg/mL for IM, intrasynovial and soft tissue injection only 10mL multi dose vial A. The dosage strength of the drug is 10 mL. B. The drug name is written using tall man lettering. C. The drug label specifies the drug is a controlled drug. D. The drug should be discarded after the first dose is administered.

B. The drug name is written using tall man lettering.

How would a drug dose that is below the minimum effective concentration (MEC) affect a patient's response to the drug? A. Drug entry exceeds drug elimination. B. The intended response fails to occur. C. The risk for toxic side effects is increased. D. The drug's duration of action is longer than expected.

B. The intended response fails to occur.

Which patient condition indicates that drug therapy for hypothyroidism is effective? A. The patient is thirsty. B. The patient's weight has been the same for 3 weeks. C. The patient's total white blood cell count is 6000 cells/mm3. D. The patient has had a daily bowel movement for the past 8 days.

B. The patient's weight has been the same for 3 weeks.

Which type of drug must always be swallowed without being chewed or crushed? A. Liquid drugs B. Time-release drugs C. Drugs that taste bad D. Drugs that act on the intestinal tract

B. Time-release drugs

What is the most important precaution to teach a patient about anti-inflammatory drugs used for respiratory problems? A. To keep the drug with them at all times to use for sudden narrowing of the airways. B. To take the drug daily, even when symptoms of airway obstruction are not present. C. To avoid crowds and people who are ill because of the greatly increased risk for infection. D. To brush their teeth at least four times daily to remove the drug from the mouth and improve taste.

B. To take the drug daily, even when symptoms of airway obstruction are not present.

Why is adenosine (Adenocard) given to a patient rapidly by intravenous (IV) push? A. If given slowly, the heart rate will dramatically increase. B. When given slowly, the drug is eliminated before it can act on the heart. C. Slow administration can lead to tissue irritation and IV infiltration. D. The drug is given rapidly so that within a minute, second or third doses may be given if needed.

B. When given slowly, the drug is eliminated before it can act on the heart.

Which unit of measurement should the nurse use to set a precise flow rate for the IV when administering IV fluids using an IV infusion pump? A. mL/day B. mL/hr C. gtt/min D. gtt/hr

B. mL/hr

Which side effects are commonly associated with most nonsteroidal anti-inflammatory drugs (NSAIDs)? (select all that apply.) A. Bleeding B. Constipation C. Drowsiness D. Dry mouth E. Gastrointestinal ulcers F. Hypertension G. Memory loss (temporary)

Bleeding, Gastrointestinal ulcers, Hypertension, Memory loss (temporary)

Which statements are true about blood pressure? (select all that apply) A. Blood pressure decreases when a person is resting. B. Blood pressure increases when a person is active. C. Low blood pressure is called hypotension. D. Systolic blood pressure is the pressure against artery walls when the heart relaxes. E. As people get older they are more likely to develop high blood pressure. F. Primary hypertension is caused by another health problem or prescribed drug.

Blood pressure decreases when a person is resting. Blood pressure increases when a person is active. Low blood pressure is called hypotension. As people get older they are more likely to develop high blood pressure.

A nursing instructor asks a student nurse, "Which insulin syringe may be used to draw 25 units of insulin?" What answer given by the student nurse indicates a need for further teaching? A. "30-unit insulin syringe" B. "50-unit insulin syringe" C. "100-unit insulin syringe" D. "Either 30-unit or 50-unit insulin syringe"

C. "100-unit insulin syringe"

A patient with asthma is prescribed albuterol (Proventil) as needed and salmeterol (Serevent) every 12 hours. When the patient asks why two inhaler drugs are needed, what is your best response? A. "Albuterol opens your airways and salmeterol decreases the inflammation." B. "I will check with the prescriber to determine whether you can just use one drug." C. "Albuterol is a rescue drug to stop asthma attacks and salmeterol prevents attacks." D. "Salmeterol helps you breathe better and albuterol opens alveoli for gas exchange."

C. "Albuterol is a rescue drug to stop asthma attacks and salmeterol prevents attacks."

What is the most important precaution to teach a patient who is prescribed a drug for insomnia? A. "Avoid drinking fluids with caffeine before bedtime." B. "Rinse your mouth frequently with water or saline." C. "Avoid driving within 8 hours of taking this drug." D. "Do not take this drug for more than 1 week."

C. "Avoid driving within 8 hours of taking this drug."

Which statement made by a patient prescribed to take antibiotic therapy for a wound infection indicates a correct understanding of the therapy? A. B. C. "Even if I feel completely well, I should take the drug exactly as prescribed until it is gone." D.

C. "Even if I feel completely well, I should take the drug exactly as prescribed until it is gone."

A patient is to be discharged to home with a continuous supplemental enteral feeding through a feeding tube. The spouse of the patient asks how much feeding to hang. What is your best response? A. "Hang enough for 12 hours." B. "Hang enough for 8 hours." C. "Hang enough for 4 hours." D. "Hang enough for 2 hours."

C. "Hang enough for 4 hours."

A student nurse was asked by the nurse what formula would be used for calculating IV intake from a primary IV infusion. Which answer given by the student nurse indicates adequate learning? A. "Hourly IV rate + Number of hours the IV fluid was infused = IV intake" B. "Hourly IV rate - Number of hours the IV fluid was infused = IV intake" C. "Hourly IV rate X Number of hours the IV fluid was infused = IV intake" D. "Hourly IV rate/Number of hours the IV fluid was infused = IV intake"

C. "Hourly IV rate X Number of hours the IV fluid was infused = IV intake"

A patient is prescribed a dry-powder inhaler (DPI) for chronic obstructive pulmonary disease (COPD). Which statement made by the patient indicates that more instruction is needed? A. "I won't exhale into the inhaler." B. "The inhaler will be kept in the drawer of my bedroom dresser." C. "I will wash the inhaler mouthpiece daily with soap and water." D. "I'll inhale twice as hard through this inhaler as I do with my aerosol inhaler."

C. "I will wash the inhaler mouthpiece daily with soap and water."

A patient is scheduled for a procedure to test mental function at 6 a.m. At 3 a.m. the patient requests something for sleep. What is your best response? A. "I'll call your prescriber and request an order for something to help you sleep." B. "Your prescriber has ordered temazepam (Restoril). I will give you a dose now." C. "I'm sorry, but there is not enough time for sleep now before your procedure." D. "I'm sorry, but you already received a dose of temazepam last evening at 9."

C. "I'm sorry, but there is not enough time for sleep now before your procedure."

The nursing instructor asked a student, "How many times will the decimal points be moved on the metric line to convert from the micro place to the milli place?" What answer given by the student indicates adequate learning? A. "Move the decimal point six times to the left, from the micro place to the milli place." B. "Move the decimal point six times to the right, from the micro place to the milli place." C. "Move the decimal point three times to the left, from the micro place to the milli place." D. "Move the decimal point three times to the right, from the micro place to the milli place."

C. "Move the decimal point three times to the left, from the micro place to the milli place."

Which statement made by a patient with heart failure indicates that more teaching is needed about the prescribed drug therapy? A. "I always try to take my heart failure drugs at the same time each day." B. "Now I am using a weekly pill dispenser to keep my drugs straight." C. "Now that my heart failure is cured I can cut back the drugs I take." D. "If I gain more than 3 lb in a week I will call my doctor."

C. "Now that my heart failure is cured I can cut back the drugs I take."

A patient tells you that after taking a daily multivitamin and mineral supplement he experiences stomach upset. What is your best advice? A. "Drink a full glass of water with your multivitamin." B. "Be sure to sit up for at least 2 hours after taking your multivitamin." C. "Take your multivitamin with food to avoid stomach upset." D. "Report this to your prescriber because a different brand might not cause this problem."

C. "Take your multivitamin with food to avoid stomach upset."

The prescriber tells a patient with allergies to use oral diphenhydramine (Benadryl) over-the-counter (OTC) to help manage her symptoms. She tells you that she would rather have a prescription for the "real" Benadryl because she knows it is stronger and will work better than the nonprescription form. What is your best response? A. "If you receive a prescription for this drug your name will be added to a controlled substances list." B. "It is better to use the OTC Benadryl rather than the prescribed form because it has fewer side effects." C. "The OTC form of Benadryl is the same strength as the one that was available by prescription only." D. "You are correct. I will ask the health care provider to write a prescription so that you can get the most effective drug."

C. "The OTC form of Benadryl is the same strength as the one that was available by prescription only."

The RN is discussing parenteral medication administration routes with the student nurse. Which statement made by the student nurse indicates a need for further learning? A. "The medication should be administered directly into the vein." B. "The medication should be administered under the skin or dermis." C. "The medication should be administered using a gastrointestinal tube." D. "The medication should be administered into the subcutaneous tissue."

C. "The medication should be administered using a gastrointestinal tube."

A 25-year-old patient is prescribed a drug for acne for 6 months that is teratogenic. Which precaution is most important to teach this patient? A. "Drugs with this classification reduce the effectiveness of oral contraceptives and you will need to switch to another form of birth control to prevent pregnancy." B. "One side effect of drugs with this classification is nausea, which will make morning sickness more severe if you become pregnant while taking this drug." C. "Use two reliable forms of birth control during the next 6 months because drugs with this classification cause severe birth defects." D. "Your risk for sexually transmitted diseases is increased while taking this drug and for 3 months after therapy is completed."

C. "Use two reliable forms of birth control during the next 6 months because drugs with this classification cause severe birth defects."

A client is receiving formula feedings via a feeding tube. The electronic medication record shows that 200 mL is to be infused every 2 hr (0600, 0800, 1000, 1200, and 1400) around the clock. The nurse gives 50 mL of water after each feeding. How much intake will the nurse record in the 0600-to-1400 shift? A. 250 mL B. 1,000 mL C. 1,250 mL D. 1,500 mL

C. 1,250 mL

A nurse receives an order for lactated Ringer's solution 1,500 mL IV to be run at 125 mL/hr. What is the infusion time for this IV? A. 8 hr B. 10 hr C. 12 hr D. 15 hr

C. 12 hr

An IV is ordered to infuse at 150 mL/hr. The nurse has an IV infusion set with a drop factor of 10 gtt/mL. What is the flow rate for this gravity IV infusion? A. 15 gtt/min B. 20 gtt/min C. 25 gtt/min D. 30 gtt/min

C. 25 gtt/min

The physician orders 1,000 mL of dextrose 5% in 9% sodium chloride (D5/0.9% NaCl) at 125 mL/hr. The nurse starts the IV at 1100. How many hours will the nurse calculate as the parenteral intake for the 0700 to 1500 shift? A. 2 B. 3 C. 4 D. 5

C. 4

For which patient should you question the order for trimethobenzamide (Tigan)? A. 70-year-old who has prostate cancer B. 50-year-old who has hypertension C. 40-year-old with a seizure disorder D. 30-year-old with asthma

C. 40-year-old with a seizure disorder

For which patient should vaccination with an attenuated vaccine be avoided? A. 18-month-old child who has asthma B. 36-year-old woman who has taken insulin by injection for 25 years C. 48-year-old man who has severe immunosuppression as a result of a stem cell transplant 1 month ago D. 66-year-old woman who takes the anti-inflammatory drug meloxicam (Mobic) daily for severe arthritis

C. 48-year-old man who has severe immunosuppression as a result of a stem cell transplant 1 month ago

How long is the minimum course of drug therapy for an immunocompetent patient with active tuberculosis? A. 7 to 10 days B. 6 weeks C. 6 months D. 2 years

C. 6 months

The IV order for a client is 900 mL D5/0.9% NaCl over 12 hours. How many milliliters per hour will the nurse set on the IV pump? A. 60 mL B. 65 mL C. 75 mL D. 94 mL

C. 75 mL

Which drug must a patient be taught to avoid during TB drug therapy? A. Aspirin B. Ibuprofen C. Acetaminophen D. Iron supplements

C. Acetaminophen

What is the most important reason intravenous potassium is considered a high alert drug? A. When intravenous potassium is administered at too low a dose its mechanism of action is greatly reduced. B. Intravenous potassium is a highly irritating solution and can cause significant pain or C. Administering too much intravenous potassium or administering it too quickly may cause life-threatening problems. D. Excessive blood potassium levels can lead to nausea, vomiting, severe diarrhea, abdominal cramping, and other intestinal problems.

C. Administering too much intravenous potassium or administering it too quickly may cause life-threatening problems.

A woman who wants to use the oral contraceptive drospirenone ethinyl estradiol (Yasmin) is prescribed to take these other drug types daily. Which drug makes Yasmin a poor choice for her? A. A corticosteroid inhaler B. A nonsteroidal anti-inflammatory drug C. An angiotensin-converting enzyme inhibitor D. A combination opioid receptor agonist/antagonist agent

C. An angiotensin-converting enzyme inhibitor

What question should you be sure to ask before administering a dose of tamsulosin (Flomax) to a patient? A. Do you have a history of high blood pressure B. Have you ever had a problem with liver disease C. Are you allergic to any of the sulfa drugs D. How often are you sexually active

C. Are you allergic to any of the sulfa drugs

How are bactericidal drugs different from bacteriostatic drugs? A. Bacteriostatic drugs are more likely to cause an allergic response than bactericidal drugs. B. Bacteriostatic drugs work only on bacteria, whereas bactericidal drugs are effective against other types of organisms. C. Bactericidal drug actions result in killing the bacteria, whereas bacteriostatic drugs only slow bacterial growth. D. Bactericidal drugs require assistance from the patient's immune system to be effective, whereas bacteriostatic drugs are effective even when function is poor.

C. Bactericidal drug actions result in killing the bacteria, whereas bacteriostatic drugs only slow bacterial growth.

A patient with indigestion is prescribed aluminum hydroxide (Amphojel). To prevent the most common side effect of this drug, what should you teach the patient? A. Take this drug 30 minutes before each meal. B. Do not drink fluids for at least 30 minutes after taking this drug C. Be sure to consume a diet with lots of vegetables and other foods with fiber D. Report any episodes of diarrhea immediately to your prescriber

C. Be sure to consume a diet with lots of vegetables and other foods with fiber

Which two conditions are possible complications of status epilepticus? A. Blurred vision and ringing in the ears B. Muscle paralysis and weakness C. Brain damage and death D. Asthma and wheezing

C. Brain damage and death

Which type of asthma problem is helped most by bronchodilator drugs? A. Thick mucus B. Mucosal swelling C. Bronchoconstriction D. Large, flabby alveoli

C. Bronchoconstriction

What should you teach any patient prescribed an antidysrhythmic drug to do every day? A. Keep accurate intake and output records. B. Drink half of a glass of water with each drug. C. Check heart rate and blood pressure. D. Participate in an aerobic exercise program.

C. Check heart rate and blood pressure.

What is the most important action or assessment to perform after giving a patient the first dose of any antiseizure drug? A. Check for gingival hyperplasia. B. Assess any IV sites and skin areas for bleeding. C. Check the patient's level of consciousness. D. Assess the patient's deep tendon reflexes.

C. Check the patient's level of consciousness.

You are administering the first dose of an opioid drug to a patient. Which specific related actions should you take before and after giving the dose? A. Ask if the patient is allergic to sulfa drugs and monitor for a reaction after giving the drug. B. Place the patient on bed rest before giving the drug and keep him or her on bed rest after to avoid accidental falling. C. Check the patient's respiratory rate and oxygen saturation before and after giving the drug and monitor for respiratory depression. D. Administer a drug such as naloxone (Narcan) both before and after giving the opioid drug to prevent respiratory depression.

C. Check the patient's respiratory rate and oxygen saturation before and after giving the drug and monitor for respiratory depression.

How do antiplatelet drugs work? A. Increasing the elimination of formed platelets B. Decreasing the formation of new platelets C. Preventing platelets from aggregating D. Trapping platelets in the spleen

C. Preventing platelets from aggregating

What is the most important action to perform before administering a pre-meal short-acting insulin to a patient with type 1 diabetes? A. Looking up the time the long-acting insulin was administered B. Assessing the body area in which the last insulin dose was injected C. Checking whether the patient's meal or snack is already on the unit D. Holding the dose if the patient's blood glucose level is under 110 mg/dL

C. Checking whether the patient's meal or snack is already on the unit

A patient taking metoprolol (Lopressor) for hypertension reports all of the following side effects. For which side effect will you notify the prescriber? A. Increased urination during the daytime B. Heart rate of 68 per minute C. Chest pain during exercise D. Decreased sexual ability

C. Chest pain during exercise

Which antiplatelet drug prevents platelet aggregation by blocking a receptor on the surface of platelets? A. Aspirin B. Acetaminophen (Tylenol) C. Clopidogrel (Plavix) D. Dipyridamole

C. Clopidogrel (Plavix)

What is the most common side effect of all drug therapy for skeletal muscle relaxants? A. Nausea B. Diarrhea C. Drowsiness D. Urinary incontinence

C. Drowsiness

Why is the intravenous route of drug administration route the most dangerous? A. If the needle clogs, the patient does not receive the drug. B. The intestinal tract does not detoxify these drugs. C. Drugs are immediately bioavailable. D. Drugs have extensive "first pass" loss.

C. Drugs are immediately bioavailable.

Which problems are the two most common side effects of drugs for insomnia? A. Increased bleeding and reduced sense of taste B. Inability to fall asleep and stay asleep C. Dry mouth and drowsiness D. Hiccoughs and diarrhea

C. Dry mouth and drowsiness

Which symptom of hyperthyroidism occurs only with the Graves' disease form of the problem? A. Goiter B. Hypertension C. Exophthalmos D. Thinning scalp hair

C. Exophthalmos

For which health problem will you specifically monitor in the older patient prescribed to take a corticosteroid for the next month? A. Sudden hair loss B. Weight loss C. High blood glucose level D. Change in sleep patterns

C. High blood glucose level

A patient with heart failure is prescribed digoxin (Lanoxin). The patient reports nausea, blurry vision, and feeling like the heart is skipping beats. What is your best action? A. Administer the scheduled dose because it will correct these heart failure symptoms. B. Assess the patient's heart rate and then administer the scheduled dose. C. Hold the dose and notify the prescriber immediately. D. Place the patient on complete bed rest.

C. Hold the dose and notify the prescriber immediately.

Which statement by a patient prescribed an antidiarrheal drug indicates the need for additional teaching? A. I will never take a double dose of my medication even if I miss a dose. B. I will get up slowly when I get out of bed or rise from a chair. C. I will contact my prescriber if the diarrhea is not relieved within 4 days D. I will avoid alcohol while taking my antidiarrheal medication.

C. I will contact my prescriber if the diarrhea is not relieved within 4 days

A patient with gastroesophageal reflux disease is prescribed ranitidine (Zantac). Which statement indicates the need for additional teaching? A. I will notify my prescriber if I need to use Zantac for more than 2 weeks B. I will look into a smoking cessation program when I go home C. I will take the Zantac first thing in the morning so that its effect will last all day and night D. I will not drive until I know how the Zantac affects me

C. I will take the Zantac first thing in the morning so that its effect will last all day and night

A woman taking oral contraceptives reports the following changes. Which one is an expected side effect and requires no intervention? A. Calf swelling B. Severe constipation C. Increased breast size D. Heavier menstrual periods

C. Increased breast size

What is the main problem of glaucoma? A. Cloudiness of the lens B. Cloudiness of the cornea C. Increased intraocular pressure D. Increased intracranial pressure

C. Increased intraocular pressure

What is the main intended action of any type of diuretic drug? A. Weight loss B. Potassium loss C. Increased urine output D. Decreased sense of thirst

C. Increased urine output

Which problem is a common side effect of insulin therapy? A. Increased blood clotting B. Decreased blood clotting C. Injection site infection D. Foot ulcer formation

C. Injection site infection

Why is more than one dose of Flumazenil (Romazicon) often needed to reverse a benzodiazepine overdose? A. It must be administered only by the intravenous route. B. It depresses the drug metabolizing effects of the liver. C. It has a shorter duration of action than most benzodiazepines. D. It must be given in low doses to avoid respiratory depression.

C. It has a shorter duration of action than most benzodiazepines.

What should the nurse understand the term "loading dose" to mean? A. It is the total amount of medication that is given in 24 hr. B. It is the dose that is necessary to withstand the desired effect of the drug. C. It is an initial large dose of medication to achieve quick therapeutic levels. D. It is the fractional portion of a total daily dose administered at stated intervals.

C. It is an initial large dose of medication to achieve quick therapeutic levels.

Which is true regarding a 1-mL syringe? A. It measures one-tenth of a milliliter (0.1 mL). B. It is most often used to prepare and administer IV medications. C. It is typically used for the administration of tuberculin skin tests. D. It is most commonly used for medications administered by all of the parenteral routes.

C. It is typically used for the administration of tuberculin skin tests.

What is the most important assessment to perform before giving the first dose of any drug to treat insomnia? A. The patient's self-reported score on a pain scale B. Heart rate and rhythm C. Level of consciousness D. Deep tendon reflexes

C. Level of consciousness

Which term means the energy use of each cell and the work A. Goiter B. Menarche C. Metabolism D. Thyrotoxicosis

C. Metabolism

Which lipid-lowering drug type can cause gout? A. Statins B. Fibrates C. Nicotinic acid agents D. Bile Acid Sequestrants

C. Nicotinic acid agents

Which action by a skeletal muscle relaxant is the major mechanism of action? A. Dilating blood vessels in skeletal muscles and reducing the need for contraction. B. Binding to opioid and serotonin receptors in the brain to induce sleep. C. Reducing motor nerve impulse transmission to the skeletal muscles. D. Inhibiting glucose uptake in skeletal muscles.

C. Reducing motor nerve impulse transmission to the skeletal muscles.

How do mucolytics help asthma and chronic obstructive lung disease (COPD)? A. Reducing inflammation B. Inducing bronchodilation C. Reducing mucus thickness D. Enhancing respiratory rate

C. Reducing mucus thickness

What is the difference between primary hypertension and secondary hypertension? A. Secondary hypertension is not as severe as primary hypertension. B. Primary hypertension usually occurs at an earlier age than secondary hypertension. C. Secondary hypertension is caused by another health problem, whereas primary hypertension does not have a known cause. D. Primary hypertension can be treated with antihypertensive drugs, whereas secondary hypertension does not respond to drug therapy.

C. Secondary hypertension is caused by another health problem, whereas primary hypertension does not have a known cause.

What makes insulin a "high-alert" drug? A. If too much insulin is given, the respiratory system is depressed. B. If too much insulin is given, the blood glucose level could climb to dangerously high levels. C. Serious harm can occur if the wrong dose is given. D. Many people are very allergic to insulin.

C. Serious harm can occur if the wrong dose is given.

How do DHT inhibitors work to reduce benign prostatic hyperplasia: A. Relaxing the muscles of the bladder. B. Increasing urine production C. Shrinking the prostate gland D. Relaxing prostate smooth muscle

C. Shrinking the prostate gland

Which unusual side effect should you teach patients and families to monitor for when eszopiclone (Lunesta) is prescribed for insomnia? A. Dizziness B. Drowsiness C. Somnambulism D. Headache

C. Somnambulism

An older adult is prescribed lansoprazole (Prevacid) 15mg orally each day. The patient asks you why a daily multivitamin has also been prescribed. What is your best response? A. As you get older you need vitamin tablets to ensure that you get the minimum daily requirements B. Lansoprazole is a proton pump inhibitor (PPI) and these drugs interfere with absorption of all vitamins from the GI tract C. Taking a daily multivitamin is important because proton pump inhibitor drugs decrease the absorption of vitamin B12 and calcium D. Older adults have more difficulty processing vitamins and minerals from the daily foods consumed

C. Taking a daily multivitamin is important because proton pump inhibitor drugs decrease the absorption of vitamin B12 and calcium

A patient taking a drug for insomnia reports becoming sunburned even when only minimally exposed to the sun. What is your best action? A. Ask the patient whether he or she usually burns when exposed to the sun. B. Instruct the patient to stop the drug and notify the prescriber immediately. C. Teach the patient to use sunscreen and wear protective clothing when outdoors. D. Document the report as the only action and reassure the patient that this problem is temporary.

C. Teach the patient to use sunscreen and wear protective clothing when outdoors.

Which side effect of fluoroquinolones is more likely to occur among older adults? A. Fever B. Hair loss C. Tendon rupture D. Low blood pressure

C. Tendon rupture

A 13-year-old boy who has a seizure disorder has grown 6 inches and gained 30 lb in the past 8 months. How do you expect these physical changes to affect his antiseizure drug therapy? A. No change will be needed. B. The dosage of the antiseizure drugs will probably need to be decreased. C. The dosage of the antiseizure drugs will probably need to be increased. D. The antiseizure drugs can be discontinued because it is likely he has outgrown the disorder.

C. The dosage of the antiseizure drugs will probably need to be increased.

A patient asks why his drug to control high blood pressure has only one generic name and two different trade names. What is your best response? A. "Most drugs have different trade names that indicate different dosages." B. "The two different trade names indicate that one is a more pure and safer drug than the other." C. The generic name is the actual official drug name and the trade name is a brand owned by a specific manufacturer." D. "If you have insurance, you can get the trade name drug, which is usually more expensive than the generic named drug."

C. The generic name is the actual official drug name and the trade name is a brand owned by a specific manufacturer."

What adjustment should you expect to be made for an 82-year-old patient who is newly prescribed chloral hydrate for insomnia? A. Seizure precautions should be implemented. B. The first dose should be given with a meal or a substantial snack. C. The initial dose should be lower than that for a younger adult. D. The patient should have continuous electrocardiography monitoring.

C. The initial dose should be lower than that for a younger adult.

Why should continuous enteral nutrition formulas be administered within 4 hours? A. When given over a period longer than 4 hours the patient's nutrition will be inadequate. B. The water soluble vitamins contained in enteral formulas quickly degrade outside of the body. C. The risk for overgrowth of infectious microorganisms in the enteral formula increases after 4 hours. D. The risk for clogging the feeding tube increases when enteral formulas are administered more slowly.

C. The risk for overgrowth of infectious microorganisms in the enteral formula increases after 4 hours.

Why is inflammation considered a nonspecific body response? A. Inflammation is the basis for almost every type of chronic disease. B. Infection by bacteria or viruses is always a part of inflammation. C. The same tissue responses occur regardless of the type of triggering event. D. The discomfort associated with inflammation can be reduced by over-the-counter drugs.

C. The same tissue responses occur regardless of the type of triggering event.

What is the name of a blood clot that forms and stays within a blood vessel or the heart? A. Antibus B. Embolus C. Thrombus D. Coagulus

C. Thrombus

What does a goiter indicate? A. Hypothyroidism B. Hyperthyroidism C. Thyroid problem D. Premature menopause

C. Thyroid problem

A patient is prescribed testosterone 100mg IM every 4 weeks. What finding will you teach the patient to report to the prescriber immediately? A. Blood pressure decrease of 10mm Hg systolic B. Urine output increase of 200ml/day C. Weight gain of 5 lbs D. Increased prostate size

C. Weight gain of 5 lbs

Which is the best clinical definition of pain? A. A state of extreme physical distress or discomfort B. A condition of sensation caused by tissue damage C. Whatever the patient says it is D. A cognitive awareness of a change in comfort

C. Whatever the patient says it is

When do most drug errors occur in a hospital setting? A. When a patient is in the emergency department B. When a patient is scheduled for a procedure C. When drugs are being administered to patients D. When two patients have the same last name

C. When drugs are being administered to patients

Which brand name oral contraceptive contains drospirenone? A. Camila B. Ovra C. Yasmin D. Zovia

C. Yasmin

Which precaution is most important to teach a patient who is newly prescribed a thyroid hormone replacement drug? A. "Avoid taking fiber supplements while on this drug." B. "Be sure to take this drug at the same time everyday." C. "Weigh yourself daily and keep a record of these weights." D. "Do not increase the dose without your prescriber's knowledge."

D. "Do not increase the dose without your prescriber's knowledge."

Which precaution is most important for you to teach a patient who has been prescribed a beta blocker drug for hypertension? A. "Avoid alcoholic beverages while taking this drug." B. "Weigh yourself daily at the same time every morning." C. "Wear gloves and other warm clothing during cold weather." D. "Do not suddenly stop taking this drug without notifying your prescriber."

D. "Do not suddenly stop taking this drug without notifying your prescriber."

A patient is prescribed levothyroxine (Synthroid) and a fiber supplement. What key point would you be sure to teach the patient about these drugs? A. "Take the drugs together as there are no interactions between them." B. "Be sure to take the fiber supplement 2 hours before a meal and the levothyroxine 3 hours after a meal." C. "Take the fiber supplement in the morning with breakfast and the levothyroxine in the evening before you go to bed." D. "Do not take the drugs together because a fiber supplement will decrease the absorption of your levothyroxine."

D. "Do not take the drugs together because a fiber supplement will decrease the absorption of your levothyroxine."

A patient taking oxycodone with acetaminophen (Percocet) for pain at home 3 days after fracturing an ankle reports constipation and continuing moderate to severe pain that is relieved by the prescribed drug. What is your best advice? A. "Either increase the time between drug doses or take only half the dose at each scheduled time." B. "Stop taking the Percocet and switch to acetaminophen alone." C. "Wrap your foot tightly and walk for at least 30 minutes daily." D. "Drink at least 3 L of fluid daily and increase fiber intake."

D. "Drink at least 3 L of fluid daily and increase fiber intake."

What is the most important precaution to teach a patient who is prescribed oral valacyclovir (Valtrex)? A. "Avoid coffee and other caffeinated beverages." B. "Wear long sleeves and a hat when going outdoors." C. "Take this drug 1 hour before or 4 hours after eating a full meal." D. "Drink at least 3 L of water daily for the entire time you are taking this drug."

D. "Drink at least 3 L of water daily for the entire time you are taking this drug."

Which dietary teaching do you include for a patient who is prescribed warfarin (Coumadin) on discharge? A. "Weight loss diets such as the Atkins diet are OK while taking this drug." B. "Be sure to eat lots of green leafy vegetables for extra vitamin K." C. "Dietary changes do not affect the action of this drug." D. "Drinking alcoholic beverages can change the action of warfarin."

D. "Drinking alcoholic beverages can change the action of warfarin."

Which statement made by a patient who is taking a thyroid replacement drug indicates that a higher dose may be needed? A. "My goiter seems to be smaller." B. "I seem to feel the heat more than other people." C. "Food just doesn't taste good without a lot of salt." D. "I am always tired, even when I get 10 or 12 hours of sleep."

D. "I am always tired, even when I get 10 or 12 hours of sleep."

What is the most important precaution to teach an older patient who is prescribed a glaucoma drug that causes miosis? A. "Wear sunglasses until the drug wears off." B. "Use at least a 30 SPF sunscreen when going outdoors." C. "Avoid driving or operating hazardous machinery until the drug wears off." D. "Increase the light indoors because vision will be decreased in low-light environments."

D. "Increase the light indoors because vision will be decreased in low-light environments."

A patient asks why she becomes sleepy after she takes a specific drug but that her neighbor who takes the same drug at the same dose does not experience sleepiness. What is your best response? A. "Your neighbor may also be taking another drug to reduce the side effects of this drug." B. "Your neighbor is probably addicted to this prescribed drug and you are not." C. "Side effects are very dependent on the time of day when the drug is taken." D. "People often experience side effects of the same drug differently."

D. "People often experience side effects of the same drug differently."

A patient who has been prescribed sitagliptin (Januvia) calls the clinic and reports swelling of the face, lips, and tongue. What is your best response? A. "Do not worry, this is a common side effect of the drug and does not require any changes." B. "Take only half the drug dose and see the prescriber within the next week." C. "Apply cold compresses to the affected areas and take an aspirin." D. "Stop taking the drug and call 911."

D. "Stop taking the drug and call 911."

A patient is prescribed ibuprofen (Advil) 200 mg four times a day as needed for inflammation and pain related to arthritis. The patient is scheduled for oral surgery. What priority teaching is appropriate for this patient? A. "The Advil will help relieve your pain after surgery." B. "Report any bruising to your prescriber immediately." C. "Ask about having the injectable form of this drug after surgery." D. "Stop taking this drug at least 36 hours before your scheduled surgery."

D. "Stop taking this drug at least 36 hours before your scheduled surgery."

A patient is prescribed to begin highly active antiretroviral therapy (HAART). What is the most important question to ask this patient before beginning therapy? A. "Do you have any symptoms now of active infection?" B. "Is there any possibility that you are pregnant?" C. "Are you currently sexually active?" D. "What other drugs do you take?"

D. "What other drugs do you take?"

A patient with heart failure reports a 5 lb weight gain during the past week and episodes of feeling more short of breath. What is your best response? A. "These changes are to be expected because you have heart failure." B. "You will probably need a decreased dose of your diuretic drug." C. "I will ask the dietitian to discuss a weight loss diet with you." D. "Your prescriber must be notified because your heart failure is getting worse."

D. "Your prescriber must be notified because your heart failure is getting worse."

The client is administered cefazolin 1 g in 100 mL (5% dextrose in water) D5W via IV piggyback (IVPB) every 8 hours at 0900, 1700, and 0100. Which is the correct value the nurse should document in the intake and output record as the IVPB intake for the 0700 to 1500 shift? A. 1 B. 1 g C. 100 D. 100 mL

D. 100 mL

A nurse finds an order for 500 mL 0.9% NaCl at 50 mL/hr in the electronic medical record. The nurse starts this IV at 1300. What is the completion time? A. 8:00 PM B. 9:00 PM C. 10:00 PM D. 11:00 PM

D. 11:00 PM

An adult client weighing 60 kg has been prescribed 180 mg of nalbuphine. What is the time period over which the medication needs to be administered at 10 mg/3 min? A. 15 minutes B. 26 minutes C. 40 minutes D. 54 minutes

D. 54 minutes

Which drop factor is found in microdrip IV infusion sets? A. 10 gtt = 1 mL B. 15 gtt = 1 mL C. 20 gtt = 1 mL D. 60 gtt = 1 mL

D. 60 gtt = 1 mL

On a 0 to 10 pain rating scale, which of the following numbers reported by a patient represents the worst pain intensity? A. 2 B. 4 C. 6 D. 8

D. 8

The nurse is recording a client's intake and output in the medical record. The client has taken in 240 mL of water with medications. How many ounces of water will the nurse record? A. 2 oz B. 4 oz C. 6 oz D. 8 oz

D. 8 oz

Which type of oral contraceptive drug must be taken every day of the month to be effective: A. An estrogen-progestin combination B. An estrogen only "mini" pill C. A progestin-drospirenone combination D. A progestin only "mini" pill

D. A progestin only "mini" pill

A nursing instructor is teaching a group of students about the routes of parenteral intake. Which statement is the instructor most likely to include while teaching? A. "It includes the IV fluid from the primary line." B. "It includes the IV fluid from the IV piggybacks." C. "It includes the IV fluid from the blood products." D. All of the above

D. All of the above

What should be included in the health-care provider's order for the administration of an IV piggyback (IVPB)? A. Dose of medication B. Time of administration C. Frequency of administration D. All of the above

D. All of the above

Which factors does the nurse consider while deciding about the concentration of medication during multiple-strength reconstitution? A. Needs of the client B. The medication order C. Route of administration D. All of the above

D. All of the above

There are two clients with acute pneumonia in the same clinical ward, and the nurse needs to administer a medication to one of them. What identifier should the nurse use to verify the right client? A. Client name B. Client date of birth C. Barcode client identification D. Any two of the above together

D. Any two of the above together

What is the best way for you to determine a patient's need for pain medication on the second day after an abdominal laparotomy? A. Check when the patient last received medication for pain. B. Assess the patient's facial expression and vital signs. C. Consider the patient's age and ethnicity. D. Ask the patient to rate his or her pain.

D. Ask the patient to rate his or her pain.

What general precaution is most important to teach a patient taking any selective immunosuppressant agent? A. Report a lightening of urine color to your prescriber immediately. B. Eat a diet that is high in fruits and vegetables. C. Drink at least 4 L of liquids every day. D. Avoid crowds and people who are ill.

D. Avoid crowds and people who are ill.

Which precaution is most important to teach an older patient who is starting histamine H2 blockers for GERD? A. Do not drink caffeine while taking this drug B. Elevate the head of your bed by 6 to 10 inches C. Take this drug with meals and before going to bed D. Avoid driving until you know how this drug affects you

D. Avoid driving until you know how this drug affects you

Which class of antifungal agents should never be given with grapefruit juice? A. Echinocandins B. Alyamines C. Polyenes D. Azoles

D. Azoles

Which health problem is the most serious possible side effect for status epilepticus? Select one: a. Ruptured spinal disks b. Brain tumor c. Broken bones d. Brain damage

D. Brain damage

A patient has a bacterial infection, but the causative organism is not known. Which type of antibacterial drug will most likely be prescribed? A. Narrow-spectrum B. Limited-spectrum C. Extended-spectrum D. Broad-spectrum

D. Broad-spectrum

Which test results indicate that a patient's antiretroviral therapy is effective? A. White blood cell count 5000 cells/mm3 B. Blood urea nitrogen level 15 mg/dL C. Blood positive for human immunodeficiency virus (HIV) antibodies D. CD4+ to CD8+ ratio increased

D. CD4+ to CD8+ ratio increased

Which assessment is most important to perform after administering the first dose of dipivefrin (Propine) to a patient with glaucoma? A. Compare urine output with fluid intake. B. Listen to breath sounds bilaterally. C. Assess level of consciousness. D. Check pupil size.

D. Check pupil size.

A patient who received 22 units of regular insulin an hour ago is now pale, sweaty, and has trembling hands, but is alert and able to talk. What is your best first action? A. Prepare to administer intravenous (IV) glucose immediately. B. Notify the prescriber or rapid response team immediately. C. Give the patient a snack containing protein and a complex carbohydrate immediately. D. Check the patient's blood glucose level immediately.

D. Check the patient's blood glucose level immediately.

Which antibiotic should be prescribed for H.pylori infection? A. Penicillin (pen V) B. Ciprofloxacin (Cipro) C. Erythromycin (Erythrocin) D. Clarithromycin (Biaxin)

D. Clarithromycin (Biaxin)

Which category of anticoagulant drugs includes warfarin (Coumadin)? A. Platelet inhibitors B. Heparin inhibitors C. Thrombin inhibitors D. Clotting factor synthesis inhibitors

D. Clotting factor synthesis inhibitors

Which problem is a major symptom of low blood sugar levels? A. Hypertension B. Flushed skin C. Weight gain D. Confusion

D. Confusion

Why are corticosteroid drugs usually prescribed for a short period of time? A. The inflammatory process cannot be controlled with less powerful drugs. B. Inhalation of corticosteroid drugs causes irreversible damage to the lungs. C. These drugs work less effectively when given over an extended period. D. Corticosteroids have many adverse effects and side effects.

D. Corticosteroids have many adverse effects and side effects.

Which term best describes what occurs when long-term opioid is suddenly stopped? A. Addiction B. Tolerance C. Withdrawal D. Dependence

D. Dependence

Which class of antihypertensive drugs has a higher incidence of side effects? A. Angiotensin-converting enzyme (ACE) inhibitors B. Calcium channel blockers C. Alpha-beta blockers D. Direct vasodilators

D. Direct vasodilators

A patient who has been prescribed an inhaled corticosteroid develops a thick white coating on the tongue and patches on the inside cheeks. What is your best action? A. Instruct the patient to stop using the inhaler until the coating is gone. B. Obtain a culture of the coating and take the patient's temperature. C. Hold the dose and notify the prescriber immediately. D. Document the finding and notify the prescriber.

D. Document the finding and notify the prescriber.

What is a common side effect of inhaled anti-inflammatory drugs for asthma? A. Anemia B. Bronchoconstriction C. Cutaneous itching D. Dry mouth

D. Dry mouth

Which side effect of ACE inhibitors is very common? A. Hypertension B. Difficulty sleeping C. Constipation D. Dry, hacking cough

D. Dry, hacking cough

Which drug belongs to the DHT inhibitor class? A. Alfuzosin B. Tamsulosin C. Silodosin D. Dutasteride

D. Dutasteride

Which hormone levels in the blood are reduced as a result of taking hormone replacement therapy for menopause? A. Estrogen and progesterone B. Estrogen and follicle stimulating hormone C. Gonadotrophin releasing hormone and progesterone D. Follicle stimulating hormone and gonadotrophin releasing hormone

D. Follicle stimulating hormone and gonadotrophin releasing hormone

What is the best way to make sure that the right patient is receiving a prescribed drug when the patient is alert and oriented? A. Ask the patient to state his or her social security number. B. Check the patient's wrist band. C. Look at the patient's chart. D. Have the patient state his or her name and birth date.

D. Have the patient state his or her name and birth date.

Which question is most important to ask a patient before giving the first dose of etaneracept (Enbrel)? A. Do you have diabetes? B. Are you pregnant or breastfeeding? C. Do you have psoriasis or another skin disorder? D. Have you ever had tuberculosis or hepatitis B or C?

D. Have you ever had tuberculosis or hepatitis B or C?

Which laboratory finding in a perimenopausal woman who takes Premarin should be reported to the prescriber: A. Random blood glucose 116mg/dl B. Potassium 4.8mEq/L C. Sodium 147 mEq/L D. Hematocrit 27%

D. Hematocrit 27%

You are checking a patient's blood pressure before administering an antihypertensive. The reading of 88/52 mm Hg. What is your best action? A. Give the patient a cup of coffee and retake the blood pressure in 30 minutes. B. Document the finding as the only action and administer the drug as usual. C. Raise the side rails and apply oxygen by mask or nasal cannula. D. Hold the dose and notify the prescriber.

D. Hold the dose and notify the prescriber.

A patient with constipation is prescribed bisacodyl (Dulcolax). The patient also states he has severe abdominal pain. What is your best action? A. Administer the drug as prescribed to relieve the constipation B. Ensure that the patient drinks a full glass of water with the drug C. Listen for bowel sounds in all four quadrants before giving the drug D. Hold the drug and contact the prescriber

D. Hold the drug and contact the prescriber

What is one of the main intended responses of fibrates? A. Increased triglycerides B. Increased blood clotting C. Increased LDL cholesterol D. Increased HDL cholesterol

D. Increased HDL cholesterol

Which mineral is critical for the thyroid to use in making thyroid hormones? A. Lithium B. Sodium C. Chloride D. Iodine

D. Iodine

What is the main feature of an opportunistic infection? A. It usually leads rapidly to sepsis. B. Children are not susceptible to this type of infection. C. It is most often caused by pathogenic microorganisms. D. It is only found in people who are immunosuppressed.

D. It is only found in people who are immunosuppressed.

What is true regarding an effective dose? A. It can be regarded as a safe dose always. B. It is the amount that is recommended in the drug reference. C. It is the amount of drug that is equal to or less than the recommended dose. D. It is the amount of drug that is enough to produce the desired therapeutic effect.

D. It is the amount of drug that is enough to produce the desired therapeutic effect.

What is true regarding a maintenance dose? A. It refers to the first dose of a drug given to a client. B. It refers to the dose of medication administered at one time. C. It refers to the minimum and maximum safe and effective doses. D. It refers to the dose required to sustain the desired effect of the drug.

D. It refers to the dose required to sustain the desired effect of the drug.

What is the priority intervention for a patient experiencing a tonic-clonic (grand mal) seizure? A. Place a padded tongue blade in the patient's mouth to prevent biting the tongue. B. Start a saline lock to administer IV antiseizure medication. C. Attach pads to the bedside rails to protect the patient from injury. D. Monitor for and keep the patient's airway open.

D. Monitor for and keep the patient's airway open.

Why should you teach a patient who is prescribed a thiazide diuretic to change positions slowly? A. Moving rapidly from a standing position to a sitting position can raise blood pressure and increase the patient's risk for a stroke. B. Moving rapidly from a standing position to a sitting position can cause excess body fluids to collect in the feet and ankles increasing the patient's risk for edema. C. Moving rapidly from a sitting position to a standing position can put pressure on the bladder and increase the patient's risk for incontinence. D. Moving rapidly from a sitting position to a standing position can cause blood pressure to drop and increase the patient's risk for falling.

D. Moving rapidly from a sitting position to a standing position can cause blood pressure to drop and increase the patient's risk for falling.

What is the difference between the multi-dose and single-dose IV vials? A. Multi-dose vials are to be administered to multiple clients, whereas single-dose vials are to be administered to a single client. B. Multi-dose vials can be stored at room temperature, whereas single-dose vials cannot be stored at room temperature. C. Multi-dose vials can be stored at room temperature, whereas single-dose vials cannot be stored at room temperature. D. Multi-dose IV vials contain antimicrobial preservatives, whereas single-dose vials do not contain antimicrobial preservatives.

D. Multi-dose IV vials contain antimicrobial preservatives, whereas single-dose vials do not contain antimicrobial preservatives.

A patient prescribed to take the oral contraceptive with the brand name Ortho Tri-Cyclen asks whether she should stop taking the pill for the 10 days that her husband is out of town on business. What is your best advice? A. No, the hormonal confusion this would cause could result in an increased risk for blood clots B. Yes, if you would normally have your period any time during the 10 days that your husband is away C. Yes, it is very important that your body experience drug-free period from time to time to ensure optimal drug effect D. No, this drug's protection against pregnancy requires taking the drug as prescribed

D. No, this drug's protection against pregnancy requires taking the drug as prescribed

A nursing instructor asks a student nurse, "Under what conditions would you interrupt the administration of IV fluid?" Which response given by the student nurse indicates the need for further teaching? A. "Infiltration of the IV" B. "Accidental removal of the IV catheter" C. "Problems with maintaining the ordered number of mL/hr" D. None of the above

D. None of the above

What information does the nurse perceive correctly per the drug label shown? **Image won't load** Dacarbazine 200mg Single-dose vial A. The strength of the drug is 19.7 mL. B. The drug has multiple dosage strengths. C. Both 1 and 2 D. None of the above

D. None of the above

How do type 1 and type 2 diabetes differ from each other? A. Type 1 diabetes develops in people under 40 years old, whereas type 2 diabetes develops only in older people. B. Type 2 diabetes develops in people under 40 years old, whereas type 1 diabetes develops only in older people. C. Patients with type 1 diabetes are at higher risk for obesity and heart disease, whereas patients with type 2 diabetes are at higher risk for strokes. D. Patients with type 1 diabetes produce no insulin, whereas patients with type 2 diabetes produce insulin but their insulin receptors are not very sensitive to it.

D. Patients with type 1 diabetes produce no insulin, whereas patients with type 2 diabetes produce insulin but their insulin receptors are not very sensitive to it.

Which term describes how the body affects drug activity? A. Drug potency B. Pharmacodynamics C. Therapeutic effect D.Pharmacokinetics

D. Pharmacokinetics

Which drug for nausea/vomiting can cause decreased sweating and increased risk for overheating? A. Meclizine (Antivert) B. Ondansetron (Zofran) C. Trimethobenzamide (Tigan) D. Prochlorperazine (Compazine)

D. Prochlorperazine (Compazine)

How do thyroid replacement drugs work to treat hypothyroidism? A. Forcing the thyroid gland to secrete more thyroid hormones B. Inhibiting the enzymes that break down thyroid hormones C. Preventing damage to thyroid endocrine cells D. Providing thyroid hormones

D. Providing thyroid hormones

Which schedule of controlled substance has the lowest potential for addiction? A. Schedule II B. Schedule III C. Schedule IV D. Schedule V

D. Schedule V

A patient who is prescribed sucralfate asks how this drug will help treat his gastric ulcer. What is your best response? A. Sucralfate decreases secretion of gastric acids to help your ulcer heal B. Sucralfate completely blocks the secretion of gastric acids so your ulcer can heal C. Sucralrfate treats the infection with H. pylori that is the major cause of gastric ulcers D. Sucralfate forms a thick coating to cover the ulcer, protect it from further damage, and allow healing

D. Sucralfate forms a thick coating to cover the ulcer, protect it from further damage, and allow healing

Which problem is a sign of an allergic or anaphylactic response to an antibacterial drug? A. Diarrhea B. Hair loss C. High blood pressure D. Swelling of the face or lips

D. Swelling of the face or lips

Which problem is a sign or symptom of an allergic or anaphylactic response to an antiviral drug? A. Nausea B. Vivid dreams C. Intolerance to fatty foods D. Swelling of the face or throat

D. Swelling of the face or throat

A patient has a deficiency of the mineral iodine. Which sign of deficiency should you teach the patient and family to report to the prescriber? A. Dehydration B. Lethargy C. Loss of appetite D. Swollen thyroid gland

D. Swollen thyroid gland

A patient prescribed prochlorperazine (Compazine) has reddish-brown urine. What is your best action? A. Collect a urine sample and send for a culture and sensitivity B. Check the patient's vital signs with special emphasis on temperature C. Assess for signs of bleeding around the urethra D. Teach the patient that this is an expected side effect of the drug.

D. Teach the patient that this is an expected side effect of the drug.

A patient prescribed an antiseizure drug tells you that his favorite morning drink is grapefruit juice. What is your best action? A. Call the diet kitchen to order grapefruit juice with the patient's breakfast tray. B. Ask the patient if he would like to have grapefruit juice added to his dinner tray. C. Instruct the patient to drink at least 8 ounces of water with each glass of grapefruit juice. D. Teach the patient to avoid grapefruit juice because it can increase the effects of these drugs.

D. Teach the patient to avoid grapefruit juice because it can increase the effects of these drugs.

Why should tetracycline drugs be avoided during pregnancy and lactation? A. Tetracycline crosses the placenta and causes brain hemorrhage in the fetus. B. The fetal and newborn liver cannot metabolize the drug and anemia results. C. The fetus and newborn are more likely to have allergic reactions to tetracycline. D. The drug interferes with tooth enamel development causing permanently stained teeth.

D. The drug interferes with tooth enamel development causing permanently stained teeth.

A 58-year-old patient with BPH asks you why he should see his physician for prostate cancer screening. What is your best response? A. BPH increases your risk for developing prostate cancer B. At your age it is always best to complete cancer screening tests C. If prostate cancer is caught early enough there is a very high cure rate D. The signs and symptoms of BPH and prostate cancer are the same

D. The signs and symptoms of BPH and prostate cancer are the same

Why is it important to always ask a patient about his or her use of any herbal supplements or botanicals? A. Many states do not have regulations about herbal supplements or botanicals. B. These substances are illegal and their use by patients must be reported. C. Patients who use botanicals seldom take their prescribed drugs. D. These substances can interact with a prescribed drug.

D. These substances can interact with a prescribed drug.

Which side effect of phenytoin (Dilantin) commonly occurs among children taking the drug? A. Delayed bone growth B. Darkening of the eyes C. Thickening of the eyelashes D. Thickening of the gum tissues

D. Thickening of the gum tissues

A patient taking tamsulosin (Flomax) asks how the drug works. What is your best response? A. This drug works on your prostate gland to decrease its size B. This drug signals the cells in your prostate gland not to grow C. This drug works by relaxing the detrusor muscle of your bladder D. This drug relaxes muscle around your urethra to improve urine flow

D. This drug relaxes muscle around your urethra to improve urine flow

Which category of drugs actually dissolves existing clots? A. Heparin B. Coagulants C. Antiplatelets D. Thrombolytics

D. Thrombolytics

A patient with severe heart failure is discharged with a prescription for nitroglycerin ointment. What technique do you teach the patient's spouse to use to avoid experiencing side effects when applying this drug? A. Squeeze the ointment onto the special paper. B. Cleanse the skin before applying the drug. C. Rotate the drug application skin sites. D. Wear a pair of disposable gloves.

D. Wear a pair of disposable gloves.

A patient with high blood lipids asks why the lipid profile did not improve after 3 months following a low-fat diet. What is your best response? A. You may need to follow a low-fat diet to improve your lipid profile. B. You must follow a low-fat diet for at least 6 months to see improvement C. You will definitely need to be prescribed a drug to see improvement. D. You may have a genetic factor that is causing your high blood lipid levels.

D. You may have a genetic factor that is causing your high blood lipid levels.

Which diuretic is potassium-sparing? A. chlorothiazide (Oretic) B. furosemide (Lasix) C. ethacrynic acid (Edecrin) D. spironolactone (Aldactone)

D. spironolactone (Aldactone)

The RN is discussing an IV lock with the student nurse. Which statements made by a student nurse indicates learning has been achieved? A. "It is a long IV line that is attached to an IV catheter." B. "It is an IV that is inserted into a vein in the hand or arm." C. "It can only be flushed with heparin to keep the IV line patent." D. "It is a small plastic tube that is inserted for the administration of fluids and medications." E. "It has an end cap that can be accessed by a needleless system to administer IV medications intermittently."

E. "It has an end cap that can be accessed by a needleless system to administer IV medications intermittently."

A nurse is teaching a group of students about drug labels. Which teaching is the nurse likely to provide? A. "The same drug can have multiple generic names." B. "Controlled substances have the potential for drug abuse." C. "The word 'capsule' in the drug label denotes a solid dosage form." D. "The brand name of the drug shows the chemical composition of the drug." E. Both b and c

E. Both b and c

A patient has been prescribed sublingual nitroglycerin. Which key points will you teach the patient about this drug? (select all that apply) A. Keep the tablet in place until it is dissolved. B. Swallow the tablet with a full glass of water. C. A tingling sensation means that the drug is potent. D. Do not eat anything until after the tablet is dissolved. E. Call your prescriber if chest pain persists after one tablet.

Keep the tablet in place until it is dissolved. A tingling sensation means that the drug is potent Do not eat anything until after the tablet is dissolved. Call your prescriber if chest pain persists after one tablet.

Which body tissues or fluids are routes of drug elimination? (select all that apply) A. Bone Marrow B. Heart C. Liver D. Lungs E. Ovary F. Saliva G. Tears

Liver, Lungs, Saliva, Tears

Which are common signs and symptoms when a patient is dehydrated? Select all that apply. A. Decreased pulse rate B. Low blood pressure C. Thirst D. Swollen eyeballs E. Dry mouth F. Skin tenting

Low blood pressure, Thirst, Dry mouth, Skin tenting

Safe drug administration requires that the individual giving a drug be knowledgeable about which drug features? (select all that apply) A. Purpose B. Actions C. Brand Name D. Side Effects E. Abnormal Reactions F. Follow-up Care

Purpose, Actions, Side effects, Abnormal reactions, Follow-up care

Which changes are intended responses of first-line antituberculosis drugs? (select all that apply) A. Sputum production is decreased. B. Cough is decreased. C. Weight is gained. D. Sputum culture for TB is negative. E. Future TB tests are negative.

Sputum production is decreased. Cough is decreased. Weight is gained. Sputum culture for TB is negative.

Which terms are used to indicate the direct IV route? Select all that apply. A. IV push B. IV infusion C. IV injection D. Bolus injection E. Continuous infusion

V push IV injection Bolus injection

Which symptoms of inflammation are caused by leakage of blood plasma into the tissues at the site of injury or invasion? (select all that apply) A. Increased production and migration of leukocytes B. Phagocytosis and fever C. Warmth and redness D. Swelling and pain

Warmth and redness, Swelling and pain

What condition is a major cause of long-term high blood pressure? A. Atherosclerosis B. Bleeding disorders C. Chronic liver disease D. Diabetes insipidus

a. Atherosclerosis

Which drug should NOT be given to children? A. prednisone B. ibuprofen (Motrin) C. aspirin (Bufferin) D. acetaminophen (Tylenol)

aspirin (Bufferin)

The parents of a child ask you why fluoride is essential for the health of their child. What is your best response? a. "Fluoride is essential for nerve health as your child grows." b. "Fluoride helps your child grow strong bones and prevents tooth decay." C. "Fluoride supports your child's immune system and prevents infections." D. "Fluoride aids your child's body in maintenance of proper fluid balance."

b. "Fluoride helps your child grow strong bones and prevents tooth decay."

Which laboratory value is always checked before giving a dose of furosemide (Lasix)? A. Calcium b. Magnesium c. Creatinine d. Potassium

d. Potassium

Which drug for high blood pressure is a beta blocker? A. Verapamil B. Telmisartan C. Captopril d. Propranolol

d. Propranolol


Conjuntos de estudio relacionados

Macroeconomics Midterm 1 - Unemployment

View Set

GRADE 10 - Biology / Digestive System

View Set

UH Manoa - PH 203 (T. Lee) - Certification Test : How To Recognize Plagiarism

View Set

15. Commercial General Liability Insurance

View Set

Sociologia applicata- corso di sociologia

View Set

Chapter 9: Violence and Serious Misbehavior

View Set